You are on page 1of 114

THE FIBONACCI QUARTERLY

THE

OFFICIAL

JOURNAL

OF

THE FIBONA CCI ASSOC I A TION

VOLUME 5

NUMBER 1

CONTENTS
P A R T I ADVANCED
ON SUMMATION FORMULAS AND IDENTITIES
FOR FIBONACCI NUMBERS
. . . . . . .

David Zeitlin

THE Q MATRIX AS A COUNTEREXAMPLE


W GROUP THEORY
. . . . . . . . .

D. A. Lind

44

ON A CERTAIN KIND OF FIBONACCI SUMS.

George Ledin, Jr.

45

LATTICE POINT SOLUTION OF THE GENERALIZED


PROBLEM OF TERQUEM AND AN EXTENSION
OF FIBONACCI NUMBERS. . . . . C. A. Church, Jr. and H. W. Gould

59

ADVANCED PROBLEMS AND SOLUTIONS

69

.Edited by Verner E. Hoggatt, Jr.

PART II ELEMENTARY
RELATIONS INVOLVING LATTICE PATHS AND CERTAIN
SEQUENCES O F INTEGERS . . . . . . . .
Douglas R. Stocks, Jr.

81

CHAINS O F EQUIVALENT FIBONACCI-WISE


TRIANGLE

87

. . . Dewey C. Duncan

ITERATED FIBONACCI AND LUCAS SUBSCRIPTS.


SUMMATION OF V ? . k m F ,
APPROACH. .
. . . .

. . .

FINITE D I F F E R E N C E
.
Brother Alfred Brousseau

ON RATIOS OF FIBONACCI AND LUCAS NUMBERS .


ELEMENTARY PROBLEMS AND SOLUTIONS

FEBRUARY

j). A. Lind

89

91

. G. F. Feernan

99

. Edited by A. P. Hillman

107

1967

THE FIBONACCI QUARTERLY


OFFICIAL ORGAN OF THE FIBONACCI
A JOURNAL
STUDY OF INTEGERS

DEVOTED TO THE
WITH SPECIAL
PROPERTIES

EDITORIAL
B r o t h e r U. Alfred
H. L. Alder
M a r j o r i e Bicknell
John L. Brown, J r .
L. C a r l i t z
H. Wc Gould
Ae P . Hillman

ASSOC I A TION

BOARD
V. E Hoggatt, J r .
Donald E. Knuth
Douglas A, Lind
C. T. Long
Leo M o s e r
I.' D. Ruggles
M. N. S. Swamy
D. E. T h o r o

WITH THE COOPERA TION OF


P . M. Anselone
T e r r y Brennan
Maxey Brooke
Paul F . Byrd
Calvin De C r a b i l l
H. W. E v e s
John H. Hal ton
R i c h a r d Ae Hayes
A. F . H o r a d a m
Dov J a r d e n
Stephen J e r b i c
R. P . Kelisky

C h a r l e s H. King
L. H. Lange
Douglas Lind
J a m e s Maxwell
S i s t e r M. de Sales McNabb
C. D. Olds
D W. Robinson
A z r i e l Rosenfeld
MQ N Sc Swamy
John E. Vinson
Lloyd Walker
C h a r l e s Rc Wall

The California M a t h e m a t i c s Council


All s u b s c r i p t i o n c o r r e s p o n d e n c e should be a d d r e s s e d to B r o t h e r U. Alfred, St.
M a r y r s College, Calif. All checks ($4.00 p e r y e a r ) should be m a d e out to the
Fibonacci Association o r the Fibonacci Q u a r t e r l y . M a n u s c r i p t s intended for
publication in the Q u a r t e r l y should be sent to V e r n e r E. Hoggatt, J r . , M a t h e m a t i c s D e p a r t m e n t , San J o s e State College, San J o s e , Calif. All m a n u s c r i p t s
should be typed, d o u b l e - s p a c e d . Drawings should be m a d e the s a m e s i z e a s
they will a p p e a r in the Q u a r t e r l y , and should be done in India ink on e i t h e r
v e l l u m o r bond p a p e r . Authors should keep a copy of the m a n u s c r i p t sent to
the e d i t o r s .
The Q u a r t e r l y i s e n t e r e d a s t h i r d - c l a s s mail at the St. Mary T s College P o s t
Office, California, a s an official publication of the Fibonacci Association.

ON SUMMATION FORMULAS AND IDENTITIES FOR FIBONACCI NUMBERS


DAVID ZEITLIN, University of Minnesota, Minneapolis, Minnesota

1.

REMARKS ON THE PAPER OF BROTHER U0 ALFRED

Alfred [ l ] has shown that


m

n-i

L1

E ^ k + r =!^\.r+L^m)

< >

k=o

+ C

* '

i=o

where C t is a constant independent of n and Ag(n) = g(n + 1) - g(n), with


A1g(n) = AfA^gCn)).

The following result yields (1.1) as a special case:

Theorem 1. Let Hn+2 = H n + I + H n , n = 0 , l , s

with H0 = p and

Hj = r. Then for n = l , 2 , - - , we have


n-i

<"> E ^ r

=H

k=o

n,rZ (?) E^^^m-s


i=o

s=o
* Hn+r+i 22
s=o

J^MJ^-MFji+iGin-s

+ Co

1=0

( r , m = 0, ! , ) ,

where
m
(1.3) C2 - - H r ^ ( - l ) i ( i I ) F 2 i G 1 m - H r + 1 5]M) 1 (i!)F 2 i + 1 G 1 m
1-0

1=0

( r , m = 0, !,)
and G

(see [2]) a r e Stirling numbers of the second kind with the properties

that Gj = 0 if i ^ 0, G\ = 1, i = 0 5 1 5 0 8 0 *
if i > s.
1

&\ = 0 if i 0, and G^= 0

ON SUMMATION FORMULAS

[Feb*

Proof of Theorem 10 We assert that


n-4
{1A)

S^k+r ^ ( " l ) W r t ^
k=o

) + C2 ,

i=o

We note that if Agfa.) = Ah(n)9 then g(n) = hfa) + C2o Thus 9 using the A operator on both sides of (1.4), we obtain

(1.5)

m
1
n m H n + r = ^ ^ ^ ^ i + n + r t A ^A,n +, I)
-xm
1=0
HI

^(-l) 1 H 2 i+n+r+i^ 1 (n in X

- /
1=0

Since fa + l ) m - n m = A(a m ), wehave A*fa + l ) m = A 1 ^ ) + A i+1 (n m )o Thus,


since Hn+2 ~ Hn+i + Hn do 5) simplifies to
m
(1.6)

n Hn+r

=^(-l)%2J+n+r+2Aj+1(iim)
3=0

'i=o

Let j + 1 = i in the first sum of (106)0 Since A

(a

) = 0, the right-hand

side sums cancels except the term for i = 09 which yields n m H

We proceed now to simplify (1.4). Since [2, p. 9]

(1.7)

A^fa) = ( - D ^ t - l ^ f 1 Jg(D + k)
k=o

we have for gfa) = n"

'

(i - 0,1, )

1967]

AND IDENTITIES FOR FIBONACCI NUMBERS


i

AV)

(1.8)

= { - l j ^ t - i ^ m & i + k) m
k=o
i

= <-'<-" (0(") k "~ v


k=o

s=o

Z(T)s<-T(-k(Ot:
m

H1~S

s=o
s=o
#

k=o

m
HI

LmmJ

S=G

since [2, p* 169, (3)]

(1.9)

(-l)1^

=X)(-Dk(k)k:

(i =

0,l,-,m)

k=o
Buschman [3, p. 6, (12)] showed that

and thus from (1.10), with s = 2i and p = n + r + l ? we obtain


(loll)

H2i+n+r+i

2i H n+r + F 2 i+iH n + r +i

Using (1.11), we obtain from (1.4)

(ia2)

n-i

k=o> % =

+
i=o

\ x{-i)iF*A%m)
H

n + r + iE ( - 1 ) i F 2i + i A i ( n m )
i=o

[Feb,

ON SUMMATION FORMULAS

If we substitute for A 1 ^ 111 ) in (1.12) by (1.8), we obtain, upon interchanging


summations,

(1.2), Add n H . to both sides of (1.2). Then, for n = 0,


n+r
all terms in the sums are 0 except for s = 0, and so we obtain C2 as given
by (1.3).
If p = 0 and q = 1, then H = F , and C2 (1.3) yields Ct in (1.1).
For calculation purposes, (1.2) is more suitable than (1.1), since Stirling numbers are tabulated. Moreover, (1.2) and (1.3) a r e in the simplest form possible*
Using& the properties
of F and G ? , we note that the coefficient of H ,
in
r
*n

n + r

(1.2) i s a polynomial in n of degree m - 1, while the coefficient of H


n+r+i
is a polynomial in n of degree m.
The following result is a generalization of Theorem Is
Theorem 2. Let
m
P(x) = V^a.x^ ,
3=
where a., j = 0, ! , , m ,
n-i
(1.13) X >
k=o

0
m '

are constants. Then for n = 1,2,, we have


m

(k)H

1)1(Xl)F

a
Z
j(sHI i=s+i

k + r = \+rJjJj4
s=o i=o

w^|2>*)'-!E'j(iHs=o
1=0

n S + C3

j=s+i

( r , m = 0 , 1 , ),
where
m
(1.14)

C3 =

-HrZ<-1)i(i:)HZa3Gj!
1=0

3=1

r+iZ(-1)i(i!)F2i+1EaJGS
i=o
(j=i
(r,m = 0, ! , )

1967]

AND IDENTITIES FOR FIBONACCI NUMBERS


Comments., If a. = 0, j = 0,1, , m - 1, and a

and (1.14) reduce to (1.2) and (1.3), respectively,.

5
= 1, then

(1.13)

A special case of (1,13)

occurs when
m
P(k) = k ( m ) = k(k - 1)* (k - m + 1) = Y V
/

kj

where (see [2, p. 142]) S-1 are Stirling numbers of the first kind. Then, since
, (m)
, /k\
,0 m
k
= m! I I , we have
n-i-i
^k(m)Hk+r

n-l
= HI! ] T P\

k=o

Hk+r

1, m

2,;-.)

k=m

Moreover, since a. = S** , Jj = 0 , l , O 8 o , m ,


j
m
m

<n = in

we have

V
a . G Sdf sm^ 3 = \ / - /.)=\l
JL*J 3 3
L
m

if
m
!
=
l
f
if i f
mj

(see [2, p. 182, (1)]). Using (1.10), we obtain from (1.14)

(1.15)

C3 - (~l) m + 1 (m!) ( F 2 m H r + F 2 m + 1 H r + 1 ) = (-l) m + 1 (m! ) H 2 m + r + 1

It should be noted that C3 in (1.14) was obtained from (1.13) for n = 0. For
P(k) = k

, the same value of C3 (1.15) is also obtained from (1.13) for

n = 0, l , - - , m - 1 (m > 1). Let P(k) = k ' m ' in (1.13), where a. = S 3 , and


let (1.13) be written as follows:

.16,

f V > H k + r - ><%
k=o

- Li(m,n)H ^ + L 2 (m,n)H ^ ^ ~ (-l) m (m!)H __, ^


n+r
.
n+r+i
2m+r+i

[Feb.

ON SUMMATION FORMULAS

We obtain f r o m (1.16)
(1.17)

vUl/

(-1)

Cm!)H 2 m +r+i = L;i(m,n)H n + r + L 2 ( m 9 n ) H n + r + 1


(n = 0 , l , * - , m ~ 1).

F r o m (1.10) with p = n + r + 1 and s = 2m - n 5


(1.18)

2m+r+i ~

2m~nHn+r

we obtain

2m+i-nHn+r+i

If we substitute for H 2:m -f r +i in (1.17) by (1.18) and then equate coefficients of


Hn+r

and H n + r + i in ( I d 7), we obtain the following i d e n t i t i e s :

m
(1.19)

(-l) (m! ) F 2 m n =

/ m

Z(-1)I(L')r E( J B) S iM

s=o i=o

(j=s+i
(n = 0, ! , , m - 1; m = 1 , 2 , - - ) ,

m
(1.20)

(-l) (m!)F2m+1n

= ^
s=o

<-i>w 2i+I P W Gji


j=s+i

i=o
(n = 0 , l ,

a , -

,m-l;

m = 1?29*).

By r e p e a t e d additions, one obtains (interchanging s u m m a t i o n s in t h e final r e s u l t )

m /

(1.21)

\H1/
(-lrCm!
)F2m+k_n = ^ T W t t )F2i+k ^ \ J ^

i=o

s=o lj=s+i

( J s ) S i G jl s

^
'

(k = 0 , 1 , ; n = 0 , 1 , v , m - 1; m = 1, 2, ) . '

Proof of T h e o r e m 20
proof of T h e o r e m 1, t h a t

Noting that A

P(n) = 0, w e find, i m i t a t i n g t h e

1967]

AND IDENTITIES FOR FIBONACCI NUMBERS

n-i
P(k)H.
k+r
jLmd

m
J ( - D ^ j + n + r + i ^P(n) + C3

k=o

1=0
- "

JLil

i=o

1=0

= H
Since

.Hi.

>(n) =

Hi

n5

X/j '

Aip(n) =

3=0

y^^

j=o

and using CL8), we have


m

]TVD^aiAW) = ^ ( - D ^ i ^ a A1^)
i=o

i=o

3=0

-E^-EvEOK""
-E^>'-EE^(H.k
1=0

j=0

i~0

S=0

S=0

* }=S

I JLmsJ

s=o

i=o

3 \^/

3"

1 i=s+i

since
m

ni

y^y^f(s 9 j) = X ^ i C f ^ ] ) a n d
j=o s=o

j - s = i f .i - s < i .

s=o j=s

The value of C3 Is obtained from (1.13) for n = 0o


2e REMARKS ON THE PAPER BY R, REICHMAN
The operator A9 where Agfa) = g(n+ 1) - g(n), is referred to as the
forward difference operator, while the operator V, where Vg(n) = g(n) - g { n - 1),
is referred to as the backward difference operator,,
(2,

J)

Indeed*

V^fa) = ]T\-1) S (Xs) g(a - s) = (-l) i ^(-l) k (i) g^'- ii + k )


s=o

k=o

ON SUMMATION FORMULAS

[Feb.

If we compare (2,1) and (1.7), we note that


(2.2)

V*gCn) = A*g(n - i)

(i - 0 , 1 , ) ;

and if g(ja) = n , we have


V%m)

(2.3)

= A*(n - i ) m

(i = 0 , l , - - - , m + l ) .

Reichman [4] gave the following results:

(2.4)

X^X 2^ ( " 1 ) i ] F n + 2 + i V i ( n m ) + 4 '

(2.5)

] T k m F 2 k = ^ ( - D ^ + M V ^ n 1 1 1 ) + C5 ,

k=o

i^o

k=o

(2o6)

i=o

^k^zk-i =^W^^iVV
k=o

1 1 1

) +C

i=o

Rao [5] generalized (2.4) and gave

(2.7)

kmH

k=o

X i (-^Hn-^+iV^n111) + cf .
i=o

The following results contain (2.4), (2.5), (2.6) and (2.7) as special cases.
The notation is consistent with Theorems 1 and 2.
Theorem 3. For n = 0, ! , ; r = 0, 1, 2, , we have

10

ON SUMMATION FORMULAS

[Feb.

vV - (.
s=o
Since

m
j=o
w e have

ra

1=0

i=Q

j=0 S=0

i=o

s=o

j=s

2}c~ir "_, Z <-S (i) i1=0

s=o

\ j=s+i

Additional simplifications a r e obtained by noting that


F

1 3 :F

~(i.-.i) " ^ ) ' - i

a+i
F . = (-1) TF .

and

T h e value of C 7 i s obtained f r o m (2.8) for n = 0.

Comments,, We note that (2*5) and (206) a r e s p e c i a l c a s e s of (208)* Suppose


now
P(k) = ( - k ) ( m )

= y^C-D^S^kJ

Since (~k)* m) = (~k)(~k-l> (~~k-m-fl) = (-.l) m k(k + 1)- (k + m - 1), we have

^(-k)(m)H2k+r
k=o

(~l)m(m!)

_^ / k + m - 1
H

2k+r

1967]

AND IDENTITIES FOR FIBONACCI NUMBERS

11

and
m

yw^G- =YsiGi = r; 5 i zm]

(2.10)

L-J

3 3

Z^ m j

[0

if

mj

Thus, from (2.9), with a0 = 0 and a. = (-1)JS J , j = l / " , m 5

we obtain

(using (1.10))
xHl

C 7 = (-l) 111 (m:)(F H - F


,KT )
m
m-i
'
m rr
m-i r+i
r+i
= -Cmi)(F H + F
H ' ) = -(ml)H ,
-m r
i-m r+i
r+i-m

(2.11)

The following result, derived via forward differences, is an alternate


form of Theorem 3, which was derived via backward differences.
Theorem 4. For n = 0 , 1 , ; r = 0, 1, 2, , we have

(2.12)

y^P(k)H 2 k+r
k=o
"m

E ( - i ) i ( i ! ) F i- 2

H<2n+r / ^
s=o

i=i

m
H

2n+r+l /

m
J

>=o 1

Eaj(Js)Gj-

J=S+1

( l)ife)r

Ei=o

( m

Hl a i(!) G i-

n S + C7

\ j=s+i
i=s+i

(m = 0,1, ) ,
where
m
(2.13)

C7 = H

a0

( 1)iw)Fi 2 a G
-X]
"
- E(j=iJ i
i=i

H +

- r iX!"
1=0

m
i)i(ii)Fi i a G 1

"E 3 3
J=i

Comments. If we compare (2.8) with (2.12), we conclude that for arbitrary a , j = 0, l , , m ,

12

ON SUMMATION FORMULAS

i=o

Ij=s+L
m

[Feb.

!
|

p-^wj .,('.)<
i=l

*j=s+i

. (s = 0 , ! , * , m - 1) ;

(2.15)

/ m

(-D^C-DWi-i E ^ S (i) G3-S

i=o

' j=s+i

'
(s = 0, !,-, m) .

For a. = S_^, j = 0,1, - , m, (2.14) and (2.15) with s = 09 yield (noting (2.10)),
respectively,

(2.16)

( - l ^ ' V l )Fm2 = ^ ( - D ^ !
i=o

(2.17)

iX] ( " 1)JS m G j

Xm = 1, 2f ) ;

j=i

m
m

)F

m
i

(-l) em!)Fm_1 = ^ ( - l ) ( i ! ) F i _ 1 ^ ( - l ) j S ^ i G J
i=o

(m = 0 , 1 , )

j=i

Addition of (2.16) and (2.17) gives


m
(2.18)

m
i

(-l) (m! ) F m _ s = ^ ( - l ) ( i ! ) F i + 1 ^ ( - l ) J S ^ G J
i=o

Since L R = F n + 1 + F

j=i

, addition of (2.17) and (2.18) gives

(m = 1,2, ) .

1967]

AND IDENTITIES FOR FIBONACCI NUMBERS


m

(2.19)

13

HI

<-l) (m2)L m 2 = 2 ( - l ) a ! ) L i ^ ( - l ) j S ^ G J
i=o

(m = l f 2, ) .

j=i

We note that (2.17) may be written as


m-i
(2.20)

m
i

=^(-D (il)Fi.1^H)iS^GJ
i=o
j=i

(nU)Fm_i[-l + (-l) ]

(m = 1,2,---) o

Thus, for m = 2n, n .= l 5 2 , - o o s (2o20) gives


2n-i
(2.21)

2n
i

(-l) (i!)Fi,1^(-l)jS^G| - 0

i=o-

(n = 1, 2, ) .

j=i

Since ([2, pp. 149, 17l])


Qm-i
b

2n

~ - G 2n
-

(2.21) may be written as

(2.22)

(2n)! (2n - l)F 2 n _ 2

2n
2n-2
= ^
(-1)^1 )Fi_ t ^
i=o
j=i

(-D^GJ

(n = 1, 2,
Suppose now
m

p(k> H k ( m ) =^]sikj
3=1

14

[Feb.

ON SUMMATION FORMULAS

in (2.12).

Noting (2.10), we obtain from (2.13)

C7 = ( - l ) m + 1 ( m ! ) ( F m _ 2 H r + F m ^ H r + i ) = ( - l ) m + 1 ( m ! ) H r + m _ i

(2.23)

If we r e w r i t e (2.12) as
n
(2.24)

yV

m )

2 k

= L*(m,n)H2n+r + L * ( m , n ) H 2 n + r + 1 + C 7

k=o
we obtain f r o m (2.24)
( - l ) m ( m ! ) H r + m 1 = Lf(m,n)H 2 n +r

(2.25)

L*(m, n ) H 2 n + r + 1

F r o m (1.10) with p = 2n + r + 1 and s = m - 2 - 2n,


H

(2.26)

r+m-i

m-2-2nH 2 n+r

(n=0,lv,m-1)

we obtain

m-i-2nH2n+r+i

If we substitute for H r + n i _ 1 in (2.25) by (2.26) and then equate coefficients of


H 2 n+r

an

d H 2 n + r + i in (2.25), we obtain the following i d e n t i t i e s :

m
(2.27)

(-l) (m!)Fm_2_2n =
s=o

i=i

I j=s+i
(n = 0 , 1 , , m - 1; m = l , 2 , ' - # )

111

(2.28)

ni

(-l) (ml)Fm_^

2n

s=o

i=o

111

''j=s+i

(n = 0 , 1 , , m - 1; m = 1, 2, ) .

Proof of T h e o r e m 4

It i s r e a d i l y verified that

1967]

AND IDENTITIES FOR FIBONACCI NUMBERS


n-i

(2,29)

15

^ P ( k ) H 2 k + r = y ^ ( - D 1 H m + r - i + i A*P(n) + C7
k=o

i=o

m
P(n) + ^ (-D 1 F i _ 2 ^Pfo)

H-2n+r

i=i
m
H g n + r + l ^ ( - 1 ) ^ 1 A*P{n) + C7

i=o

since H 2n + r _i+i = F ^ ^ n + r

Fi_iH2n+r+i5

which is obtained from (110)

where s = i - 2 and p = 2n + r + 10 The simplification of (2.29) to the form


(2*12) proceeds in the same manner as in the proof of Theorem 2.

The value

of C7 (2.13) is obtained from (2.12) for n = 0.


The following result, derived via backward differences, is an alternate
form of Theorem 2, which was derived via forward differences.

(2.30)

n
y ^ P ( k ) H*k+r
]

Since

= y Y l ) X ^ r + 2 4 j V W ) + C3

k=o
m
P(n) + ^ < - l ) i F i + 1 V i P ( i i )

- H,

i=i
m
+ H a + r + ^ ^ D V i ^ V ^ f c i ) + C3
i=o
we may now state
Theorem 5. For m = 0 5 l 9 e o o ; n = l 9 2 5 c
n-i

(2.31) X >

)H

k + r = n + r E C"1)

k=o

s=o

E,!'"|E<-uS(l)Qi1-

m
B-n+r+i 2 , ^~1^
s=o
+ C3

I j=s+i
"m

| m
I j = S+l

s v

16

[Feb.

ON SUMMATION FORMULAS

where
m

m
(2,32)

3 =

{ 1)Ja G

" r] i+i Z "


i=i

j J

- ^E

Ij=L

i!F

i + 2 Z(-1)JajGJ

1-0

\ j=i

(r, m = 0,1, )
Comments,, If we compare (1,13) with (231), we conclude that for arbit r a r y aJf

J = 0 , 1 , . . . ,m ,

(2.33)

' '

:
s+i
>" j=s+i

)
m

<- Z"< E . <-l,S(i)GJj=s+i

(s = 0,1, , m - 1) ;
m
(2.34)

XM^OF^J
i=o

a/nGH

I j=s+i

^ '

'

= (-i) 5>Fi+2 j=s+i


x;_ <-s (i) <
(s = 0, ! , , m)
For a. = (-1) J S^, j = 0, l , - - , m ,
(2.10)), respectively

(2.35)

(2.33) and (2.34) with s = 0, yield (noting

m
m
m ! F m + 1 = (-!)*(!! )F 2 i
i=l

(-D^G*

J=i

Cm = 1, 2, ) ;

1967]

AND IDENTITIES FOR FIBONACCI NUMBERS


m

(2.36)

17

m
1

ml F m + 2 = J^ {-I) *!! )F 2 i + 1 ] T ^SLG]


i=o
j=i

(m = 0 , 1 , ) .

Suppose now
m
(m)

P(k) = (-k)

= S(-D 5 s4k j

in (2.31). Then
n-i

n-i
/ k + m - 1\

k=o

T-

k=i

k+r

'

and from (2,32) we obtain


(2.37)

C3 = - ( m ! ) ( F m + 1 H r + F m + 2 H r + 1 ) - - ( m I ) H m + r + 2
We note that (2.4) and (2.7). are special cases of (2.30)..
3. ADDITIONAL RESULTS
In terms of forward differences it is readily verified that
n-i

(3.1)

m
p k H

J2 < > 3k+r


k=o

J^ ^ 1 ) l 2 " 1 " l H 3n+r>l+2i ^ W


i=o

C8

m
3 n + r ^ (-l) i 2- i " 1 F 2 i _ 2 A^tn)
i=o
m
+ H3I1+r+i 2
1=0

(- 1 ) 1 2 " 1 ^ i - i ^ ( D ) + C8

18

ON SUMMATION FORMULAS

M o r e o v e r 5 in t e r m s of b a c k w a r d differences, it i s r e a d i l y verified that

(3.2)

2 POOHsk+r

m
] T (-1) X 2

k=o

i=o

^n+r^iV'PW

+ C8

m
H3n+r^

(-DV^^^iV'pW

i=o
m
+

3 n + r + i J ] (-l) i 2- 1 "" i F 2 i V 1 P(n) + C 8


1=0

The following r e s u l t i s a r e s t a t e m e n t of (3.1) and (3.2):


T h e o r e m 6.

For n = 1,2,; r = 0,1,2,",

n-i
(3

3)

m
p(k)H

X)
k=o

3k+r

we have
|

( - D W - V , a. (J)G

sn+rS

i=o

s=o

'i=s+i
J=S+1
m

m
+ H 3 n + r + i 2^
s=o

'j=s+i

1=0

(m - 0 , 1 , )
where

(3.4)

C8 = -H r Y, C-Di(i02"i"1F2i2

a G

i S

J=i

1=0

"

Hr+1

E (-D (i02- - F2i-1 J ] ajGi


1=0

i 1

J = l

F o r n = 0 3 1 , *; r = 0, 1 , 2, * , we have

1967]

AND IDENTITIES FOR FIBONACCI NUMBERS


HI

(3.5)

]Tp<k)H 3k+r = H 3 n + r ] T (~1)


k=o

m
lj=s+i

s=o

+ C8

I m

"HI

+ H.3n+r+i

19

Es=o <-'

^j=s+i

(m = 0, ! , - )

where

HI

m
(3.6)

I>>S G J

C8 = H r

3=1

i=o

/ HI

HI

^iZ^^VE'-^f!
J 3

1=0

3=i

Comments,, Add P ( n ) H 3 n + r t o both sides of (3.3), Then, c o m p a r i n g (3.3)


and (3.5), w e conclude that for a r b i t r a r y a., j = 0, 1, - , m ,

(3.7)

a s + J ] (-l)i(iI)2-i-1F2i_2
i=o

a ^ G *
I j=s+i
i=s+i
m

^ T f % E (-S(lK
\ j=s+i

1=0

(s = 0 , 1 , , m ) ;
m

(3.8) j ; w w % j ; a lu;
1

1=0

HI

j=S+l

/HI

( _ D ^ ^ - - V i E <1=0

li=s+i

1)Ja

j(i) 3-s

(s

= o.i.-.>

20

[Feb,,

ON SUMMATION FORMULAS

F o r a. = S^ ,

j - 0 , 1 , - , m,

(3.7) and (3.8) with s - 0, yield (noting (2.10)),

respectively

(3.9)

m! 2-m"1F2m_2 = J ] i! 2"i"1F1-i ]T t - D ^ G J
1=0

(m = 1, 2, ) ,

3=i

(3.10)

m!2

hf^-i

il 2

^ - i E

1=0

( 1)Js G
" m J

<m = 0,1, ) ,

13=i
I

which m a y b e simplified by noting that Fi_j = (-1) F i _ i and F 2 _i = (-1)


If a. = ( - l ) V , j = 0 , 1 , , m , (3.7) and (3.8) with s = 0
respectively,

(3.11)

>l
Fi_ 2 yield,

(ml)2~m V J
m
i

^C^i) (i:)2" -Si-2E(-1)JsmGj


[

1=0

(3.12)

-(m!)2

(m = 1 2 o o o )

' '

3=i

^m^a
m

= ^(-l)i(i!)2-i-1F2i_ii;(-l)JsiGi
i=o

(m = 0 , 1 , )

3=i

By r e p e a t e d additions, (309) and (3.10), a s well a s (3.11) and (3.12), give s i m i l a r


identities for Lucas n u m b e r s ,

L .
n

Suppose now

po

(m)
s k

= 2 s m kj
j=i

1967]
in (3o3)

Then, from (3.4), we obtain


C 8 = ( - l ) m + 1 ( m S )2

H1 1

- (F2m_2Hr + F2m_iHr+1)

F r o m (1.10) with p = 3n + r + 1 and s = 2m - 2 - 3n,

Efem+r-l ~

2m-2-3nH3n+r

If we s u b s t i t u t e for
H3n+r+i,

(3.13)

21

AND IDENTITIES FOR FIBONACCI NUMBERS

we obtain

2m-l-3n H 3n+r+i

C 8 in (3.3) and then equate coefficients of

H3n+r

and

we obtain the following i d e n t i t i e s :

(-l)m(ml )2-m"1F2m_23n
m

-z

s=o

i=o

'j=s+i
(n = 0 5 1 , , m - 1; m = 1, 2, ) ,

(3.14)

(-l)m(m!)2

"1F2m-l-3n
m

E
s=o

Ij=s+i

1=0

(n = 0, V n i . - l ; m = l , 2 , - - )

Suppose now
m

P(k) = (-k) (m) =


3=i

<-l)jS^

22

ON SUMMATION FORMULAS

in (3.5).

[Feb.

Then from (3.6), we obtain

C 8 = - ( i n i ) 2 ~ m ~ i ( F 1 _ 1 I 1 H r + F 2 _ m H r + 1 ) = -(m!)2~ m " 1 H r -f2-m

4.

GENERALIZATIONS

Let a, b, U 0 and Ui b e a r b i t r a r y r e a l n u m b e r s , and c o n s i d e r the following t h r e e s e q u e n c e s :


(4.1)

Un+2 -

a U n + 1 + bU n ,

(4.2)

U n + 2 = aUn+i + U n ,

(4.3)

Un+2 = Un+i + bUn ,

ab = 1,

a f

-19

(n = 0 , 1 , - - )

a f

0,

(n = 0 , 1 , - - ) ,

b = 0,

(n = 0 , 1 , - )

, .

We note that (4.1), (4.2), and (4.3) r e d u c e to the F i b o n a c c i sequence for t h e p r o p e r


c h o i c e s of a and b We shall obtain s u m m a t i o n f o r m u l a s , u s i n g both forward
and backward differences, for each of the t h r e e s e q u e n c e s , a s defined by (41),
(4.2), and (4.3), which yield the p r e v i o u s r e s u l t s , i . e . , T h e o r e m s 2, 3, 4, 5,
and 6, a s s p e c i a l c a s e s for the p r o p e r choices of a and b

We have a l r e a d y

s e e n how c e r t a i n p r o c e d u r e s m a y b e u s e d to obtain v a r i o u s identities from our


T h e o r e m s 2, , 6 In view of s p a c e l i m i t a t i o n s , no attempt will b e m a d e to
u s e t h e s e p r o c e d u r e s to fully exploit t h e g e n e r a l r e s u l t s obtained in t h i s section B
Identities given in t h e proofs of T h e o r e m s 2 and 3 will be u s e d to obtain t h e
explicit f o r m u l a s cited in our g e n e r a l t h e o r e m s , whose proofs a r e s i m i l a r to
that u s e d for T h e o r e m 2 (if f o r w a r d differences a r e involved) o r to that u s e d
for T h e o r e m 3 (if b a c k w a r d differences a r e involved). We shall u s e r e p e a t e d l y
t h e following identity [3, p . 6, 12]

1967]

AND IDENTITIES FOR FIBONACCI NUMBERS

(4.4)

Up+ S - b ^ g U p - ! + * s + i U p

where $0

0> <i = 1,

and $ n + 2 = a$n+i

(4.4) yields (1,10) for a = b = 1.


p a r a m e t e r range 9
before,,

P(k) (see T h e o r e m 2) is defined

U - n .= (U 0 V n - U n ) / ( - b ) n
V 0 = 2,

Vi = a,
n

n-i
S

(n = l s 2 5 )

and V n + 2 = aV n +i + bV n ?

n = 05l,e" ,

Since

3k+r

= J^i-DV

+ b r ^ n + r - i - h s i ^ P ( n ) + C^

i=o
m
+ b ) " 1 " 1 ^ ^ A2P(n)

- bU3n+r ^ ( - D V
i=o
m

+U8n+r+i^(l)i(a2 + W

" ^ . ! A W ) + C8*

1=0

and
HI

<*)

as

We note

P{k)U

k=o

4 7

that

n = l,2,-- .

(i) Let U n satisfy (4.1).

<

fofai* n = 0 9 1 9 . We note

All r e s u l t s in this section a r e valid for t h e

r = 05 1 , 2 5 .

that $ _ n - - * n / { - b )

4 6)

F o r negative s u b s c r i p t s , we define

(4.5)
where

23

k=o

3k+r

= ^(-D^a

+ b)

h l s n + ^ - i ^ P C n ) + Cs

i=o
m
= bIJ

3 n + r S (""1)1(a2
i=o
m

1=0

W'^Wi^Pdi)

24

[Feb.

ON SUMMATION FORMULAS

We may now state


Theorem 7. Let U n satisfy (4.1). For n = 1, 2, , and m = 0 , 1 , " # ,
we have
n-i
(4.8)

y^P(k)U3k+r
k=o
m

bU3n+r V "
s=o
+

^(-lftilMa* + b ) " 1 - ^ J^
>
m "m

j (Js) GjL

'j=s+i
/ m
n

3n+r+i / j
s=o

C ft

lj=s+i

1=0

where
m
m
2
i 1
C* = - b U r ^ ^ D ^ i l X a + b)" " 2 i-2 E a j

(4.9)

1=0

/ m
i

U r+1 ^(-l) (il)(a + W ^ W i J ^ a . G J


1=0

and m = 0 , 1 , o o , we have

For n = 0 , 1 , ,

(4.10)

J=i

n
^P(k)U3k+I
k=o
"m

m
= bU

( m

1 1

^ 1)Ja j (3s) G j -

to+r]<-l> ] T i : (a* + b)" " ^! J^


s=o

i=o

m
u

3n+r+i J> (- If
s=o

j=s+i
/ m

^ i i ( a 2 + w - ^ W i J ] ("1)Ja5 (i) G j^
1=0

i=s+i

+ C8 ,

1967]

AND IDENTITIES FOR FIBONACCI NUMBERS

25

where

(4.11)

Ca - U

" b^ii(a2+b) i i ^ i - i ly^ ( - i)3a - Gi l


i=o

( j=i
| m

- ur+iX/Ma2

+ b)

)_
\

" i "^ 2 - i E ( " 1)Ja j G j

1=0

'

3=i

For a = b = 1, Theorem 7 yields Theorem 60


(ii) Let U

(4.12)

n-i
^PU

satisfy (4.2). Since

2 k + r

m
= J ] ( - l ) i a " i ' 1 U 2 n + r . 1 + i A i P ( n ) + C?

k=o

1=0

U2n+r

P(n)

+ ^ ( . D V

"

^Pfo)

i=i

m
U g n + r + i ^ - l ) ^ " 1 " 1 ^ - ! A*P(n) + C?
1=0

and

(4.13)

n
^P(k)U2fcfr
k=o

m
^ ( - 1 ) l a " 1 " l u 2 n + r + i - i V 1 P ( n ) + C*
i=o
U^+r^VDV

*_ i V 1 P(n)

i=o
m
+ U a a + r + i ^ t - D V ^ U i - i ^ P f c ) + C7* ,
1=0

we may now state

26

[Feb.

ON SUMMATION FORMULAS
Theorem 8. Let U

satisfy (4*2), For n3 m = 0 5 1 3 *, we have

XX

(4.14)

P(k)U 2k+I

k=o

m "m
m
=

2n+r /

s=o i=i
m I m
u
2n+r+iy J
s=o

in

3=s+i
j m
n

+ C7

*j=s+i

1=0

where
m

(4.15)

C* = UT ~ u r a0

E^^^-E^i
3=i

i=i

HI

" Ur+iS^^tt^^Vi^ajGJ
1=0

]=i

For n, m = 0,1 ? - , we have

(4.16)

y^P(k)U2k+r
k=o

m
(-1)
s=o

E'-^-'lEw'Sli)
i=0

+ c* ,

< s

| j= s +i

"m

( m

+ U2n+r+i/J(-DS
s=o

1=0

I j=s+i

ns

Mo-' ll
3W

3-s

1967]

AND IDENTITIES FOR FIBONACCI NUMBERS

27

where

(4.17)

C* - U7 - ur
1=0

I j=i
I in

Ur+i

X / a " i " 1 ^ i ~ i ( " i)Ja j G j

1=0

For a = 1, (4.14) and (4.15) yield Theorem 4; and (4,16) and (4017) yield
Theorem 3.
(iii) Let U n satisfy (4e3)

(4018)

Since

n-1
m
^P(k)Uk+r = ^ ( - 1 ) ^
k=o

Hln+r+i^i ^Pfo) + C*

1=0

b U n + r ^ ( - l ) V 1 fy2i ^ P W
i=o
m
+ U n + r + 1 ^ ( - l ) V 1 " V 2 i + i ^ P ( n ) + C*
1=0

and
m

(4.19)

^ P ( k ) U k + r = ^(-Dhr1
k=o

V + r + a + i V ^ W + Cj

1=0

= bUn+r

b 1]P(n) + ( 1 ) J b i 1 i + l V i p & l )

"" X/ " " " *


i=i

m
Un+r+i^jM-l)^
1=0

V i + a ^ P W + C* ,

28

[Feb.

ON SUMMATION FORMULAS

We may now state


Theorem 9. Let U

satisfy (4 3). For m = 0, ! , ; n = 1,2,-**,

we

have
n-i

(4o20)

y^P(k)Uk+r
k=o

- bUn+r

m Tm

.
m

=oli=o
s^o[i=o
m
m fTni

'j=s+i
( m

+ Un+r+1 ^ ^ ^
s^o I i=o

s , 0*
+ C3 ,

/ m

3=s+i

where
m

(4.21)

C* = -bUr^C-lJ^Ob-^WEa.Gjj
i=o

( j=i

"

Ur+1

/ m
( 1)i(il

]C "

^"^^i+i 5^ j J
3=i

For m = 0 , ! , ; n = 1,2, ,
n-i
fc=o

+r "

bu

n + r s=o
^ ( " 1 ) " i=i
u

m
n+r+l / ^(-D
s=o

+ c3
where

we have

]TW)U k

J
a G

i=o

(4,22)

(j=s+i

i~o

I j=s+i

\
'

1967]

AND IDENTITIES FOR FIBONACCI NUMBERS


m

(4.23)

j m
1

C3

-bUry^ilb" '"Vi+i
i=i

\
j

\^(-l) a.G!
\ j=i

J
m

, m

1=0

F o r b = 1,

29

]=i

(4.20) and (4.21) yield T h e o r e m 2; and (4022) and

(423)

yield T h e o r e m 5e

5.

APPLICATIONS FOR A SUMMATION FORMULA

Recently, t h e author [6] p r o v e d t h e following r e s u l t "


L e m m a 1 Let u . , i = 0 , 1 , , p - 1, be a r b i t r a r y r e a l n u m b e r s ,
let u , n = 0 , 1 , ,

o r d e r p with r e a l , constant coefficients e

(5.1)

oU n +p + b 1 u n + p - i + 8 8 + b p u n = 0

Let x b e a r e a l number*

(5.2)

3-i

2 Ev -E B
1=0

k=o

(b 0 b p 0)

Then

bfxi

and

satisfy a homogeneous, l i n e a r difference equation of

bpn+i+k-j

k=o

i=o

p-i

k=0

j=o

n+i+k

30

ON SUMMATION FORMULAS

[Feb.

Tb.u, . :
(5.3)

> u,x
k

k=o

i=o

The series in (5.3) converges for | x | < I XI, where A is the root of bpxP +
+ bjx + b 0 = 0 with the smallest absolute value.
In [ 6 ] , (5.2) was used to obtain a closed form for

E*

k=o

If x0 is a value of x such that

Jji4 - >
i=o

then

^
k
ukx0

k=o

is obtained from (5.2) by applying LTHospital1 s rule.


As before, let
m

P(k) - y \ . k j ,

f 0 ,

j=o

and consider u, = P(k)w ,


2,--,

and

, k = 0 , 1 , , where q = 1, 2, ; r = 0, 1,

1967]
(5.4)

AND IDENTrriES FOR FIBONACCI NUMBERS

31

w n + 2 + diWn+j + d2wn - 0, d ^ 0, d2 - 4d2 ^ 0, (n = 0 , 1 , - . . ) .

If a and /3 are the roots of x2 + dix + do = 0, then IT, = w , , satisfies


1
*
k
qk+r
(5.5)
since

Uk+2 - VqUk+1 + d f u k = 0
(x - afl)(x - j8Q) - x2 - Vqx + d,

(k = 0 , 1 , - . . ) ,
- cP + 0 n , n = 0 , 1 , %

where V

with VQ ='2, Vi = -d l 5 satisfies (5e4)8 We note that P(k)wq k + r is a solution


of a homogeneous, linear difference equation of order 2m + 2 with real, constant coefficients whose characteristic equation is given by
(5.6)

[(x - aq)(x - j 8 q ) ] m + 1 = (x2 - Vqx + df)m+i

- 0 .

Since
2m+2
2

(x - Vqx + 4 )

m+1

b2m+2_sxS

- ^
s=o

we have that

m+1

(i - v qX + dfx^)

2m+2

y.3

j=0

In [2, p* 30, example 3], it is shown that


m+i

(5.7)

b. = ( - ^ ^ ( ' ^ ^ ^ ( j - i ) ^
i=o

" ^ ^

CJ = o . V . 2

Thus, (5.2), in which p = 2m + 2 and b. defined by (5.7), yields a closed form


for

2
k=o

k
Pfl^w^^x
v
qk+r

32

ON SUMMATION FORMULAS
If w, = KL, then dt = d2 = - 1 , V q = L q 5

(5.8)

-(1 - Lqx +

[Feb.

and (5.2) yields

(-1)^)m+^p(k)Hqk+rXl5
k=o

2m+i
^b.Pfo

+ 1 + k -

j)Hq(n+1+k_j)+r x

n+l+k

k=o
2m+i

k
^ b . P ( k - j)Hq(k-j)+r x

k=o

(n = 0 , 1 / " )

3=o

w h e r e (see (5.7))
m+i
(5.9)

2 m + 2).

If P(k) = k

= ml l ^ J

in (5.8), we conclude that for a r b i t r a r y

x,

2m+i
(5.10)

XX

mk

~J)

n+i+k
q(n+i+k-jHr

k=o
2m+i
H

k=o

j=o

q(k-j)+r

'
(n = 0 , 1 , , m - 1; m = 1, 2, )
OTYI-J-0

If n = 0 in (5.10), the coefficient of x

21X1+1

(5.11)

m u s t b e 0,

/
\
f 2m + 2 - j l H
= 0
3\
m
/ q(2m+2-j)+r

i. e.

(m = 1 , 2 , - . . )

1967]

AND IDENTITIES FOR FIBONACCI NUMBERS


If P(k) = ( - k ) ( m ) - ( - l ) m ( m ! ) / k

33

" A in (5 8 8) 3 we conclude that for

a r b i t r a r y x and n = 0 5
2H1+1

(5.12)

V *
k=o

. i+k

|Hq(i+k-j)+r

3=0
2H1+1

x*

(m-= 1 , 2 , - - - )

j=0

k=o
In (5.12), t h e coefficient of x2m+2 m u s t be 05 i. e . ,
2K1+1
{5 13)

J2
i=o

,
b

j (

v
3 m

m 1 " j ) Hqfem-w-jHr

(m -

1,2,---)

If P(k) = 1, then (5.8) yields a r e s u l t which h a s a l r e a d y b e e n p r o v e d by t h e


author [ 7 , p . 105 9 ( 5 ) ] s u s i n g a different procedure*
Noting t h a t w m = c o s md and w n = sin n# satisfy w n + 2 - 2 cos 0w n -n +
w n = 0, n = 0 5 1 , 9

with V n = 2 c o s n 0 ,

where

0 fi 09TT,

0 < 0 < 2rr9

we obtain f r o m (5.2) t h e following two i d e n t i t i e s :

(5.14, - [ ! - 2(003 q x + *T+'2>{r S:3J}'


k=o
2m+i

b.P(n
k=o

k-

)|

s i n

. n+i+k
rq(n + l

k - j ) + r ] 4

J^o

2m+i
k=o

Z^
]
3=0

P ( k

M s m [ q f e - j) + r ] 0 /

(n = 0 , 1 , )

w h e r e {see (5.7))
1X1+1

(5.15)

b. = <-!>J J ] ( m ^ / ( j - i ) (2 C S qe)2i_j
i=o

(j =

' 1 '"*' 2m+2) -

34

ON SUMMATION FORMULAS

[Feb.

The relative simplicity of our results, (5.14) and (5.15), may be compared with
the less general (as well as less elegant) results obtained by Schwatt [8,
pp0 217-219], who used the differential operator,
For choices of P{k) = k

or (~k)

(xd/dx)

, we obtain (in the same manner

as (511) and (5.13)) the identities (pairwise)


2m+i

(5 16)

Y^
LJ

(5ol6)

b ( 2m + 2 - j W cos [q(2m + 2 - j ) + r ] ^
j \
m
/isln[q(2m+2-j) + r]f

{Jl

2H1+1

(5.17)

y
JLJ

j=o

b . ( 3 m + i - j M c o s [ r q g m : ' ^ ! : r ] / J = o <m = i.2.->


3\

ni

y i sin [q(2m + 2 - j) + r ] 0 J

Identities (5016) and {5.17) may be transformed to hold for hyperbolic functions
by recalling that cosh (i#) = cos 0 and sinh (10) = i sin# e
As an application of (5.3), we have

(5.18)

(1 - Vqx + dfx2 ) m + i y ^ P(k)w q k + r x k


k=o
2m+i *~"

2 ^ b , P ( k - j)w q (k-j)+r x
k=0

3=0

where b. is defined by (5.7).


It is desirable to have check formulas for the computed values of b.. In
our discussion, consider b., as given by (5.7), where

(5.19)

2m+2
(1 - Vqx + d f x 2 ) m + 1 - ^
b.x j
3=0

(m = 0 , 1 , - - - ) .

We may set x = 1 in (5.19). A substantial reduction in the effort required


to evaluate all the b., j = 0 , 1 , a , 2m + 2, is afforded by noting that

1987]

AND IDENTITIES FOR FIBONACCI NUMBERS


b ^ . j = d?(m+H)b.

(5.20)

(j = 0 , 1 , . . . , m + 1) .

To prove (5,20), multiply both sides of (5.19) by d ^ m + l ) ,

(5.21)

(d? - d2Vqx + d f ^ )

m+1

35

and so

2m+2

= J2 b j ^ ( m + l ) x j

5=o
Replacing x in (5.21) by x/dj?, we obtain (in reverse order)
2H1+2

m+1

(5.22)

(* - Vqx + d^)

2TCL+2

= J^ Y^ "^

b2m+

HxJ

and thus (5.20) is obtained by comparing the coefficients of x** in the sums in
(5.22).
Let t = 1 , 2 , . ' . ,

and let gt+i(x) = 0 (where gt+i(x) is a polynomial in

x of degree t + 1) be the characteristic equation determined by H Q ^- +r . Then


the characteristic equation determined by u,
x

[gt-Hl( )]

= P(k)H q ^. +r

is given by

Since

t+

m+i

(t+i)(m+i)

[x wa/x)] = Y, v j
j=0

(5.2) may be applied to yield a closed form for

EP(k)Hqk+rxk
k=o

A formidable obstacle in this procedure is the complex nature of the b . , which


involve multiple summations.

36

ON SUMMATION FORMULAS

[Feb.

As a simple example, consider H ^ + r , where H ^ + 3 + r - 2 H ^ + 2 + r - 2 H ^ + 1 + r


+ H

and g 3 (x) - x 3 - 2x 2 - 2x + 1.

= 0,

Then x 3 g ( l / x ) = 1 - 2x - 2x 2 + x 3

and
3<m+i)
(1 - 2x - 2x 2 + i?)m+1

b x3

= ^

i=o
Using the binomial t h e o r e m and then applying (5. 7) (with t h e p r o p e r change of
notation for t h e coefficients) , w e obtain
m+i
(1 - 2x - 2x 2 + x 3 ) m + 1 = X K

+ 1

( W C l + x -

(K^/2)]1

i=o
m+i i

2i

3m+3

-Zm^Zv^-EV
i=o

'

k=o

j=o

where
i

E(s)( t - s 0 ( - 1/2)t

>

U I i i,

,k-s
U-V*J

(k = 0, ! , , 2i) ,

s=o

and

(5.23)

b. =

f 1 *) (-2)^.1

1=0
m+i
=

i=o

/ s=o
(j = 0 , 1 , - , 3m + 3)

T h u s , from (5.2) with p = 3m + 3 and u, = P(k)H 2


is defined by (5.23)),

we obtain (where b .

1967]

AND IDENTITIES FOR FIBONACCI NUMBERS

- ( 1 - 2x - 2x 2 +

(5,24)

x 3

>m+iy]p(k)H|
k+r
k=o

3m+2
k=o

37

i Y ^ b . P ( n + 1 + k - j)H*
n+i+k-j+r
J=o

n+i+k

3m+2

X>Jp*-J)HLjj+r
J=o

k=0

Recalling t h e m a n n e r by which (5.11), (5.13), (5.16), and (5.17) w e r e


obtained, we m a y now s t a t e t h e following r e s u l t :
T h e o r e m 10. Let
(t+i)(m+i)
(5.25)

[x*

g t f j d / x ) ]im+1

b.x]

(m = 1,2,- )

F
Then

(5.26)

(t+i)(m+i)-l
y

/(t

l)(m +

l)-j)

i
q(tm+t+m+i-j)+r

= 0

j=o

(q,t,m = 1,2,---;

r = 0, 1 , 2, J

(t+i)(m+i)-i
(5.27)

J=o

-i(*

+ l)(m+l) - j
m

(q,t,m = 1,2,---;

1 + m\

r -

t
q(tm+t+m+l-j)+r

0, 1 , 2, ) .

We note that (5.26) and (5.27) a r e identical for m = 1.

3S

ON SUMMATION FORMULAS

[Feb.

60 REMARKS ON THE PAPER BY LEDIN [ 9]


From our (2.31) with r = 0, EL = F , , and P(k) = k m (so that a
a. = 0,

j = 0 , ! , 5m - 1),

=1,

we conclude (see 9, (3a), (3b) for notation)

that

3
(6sl)

i*3 Z

kIF

k+iGf

= 0 1

' '")

k=o

2,j = Z k l F k + 2 G ^

0 = 0,1,-

k=o

From [9, (6a)], we obtain for i = 3

(6 3)

3,3 =JjlF^

'

"j

t^0'1'-)

k=o
Thus, the assertion [9, (6e)] is valid only for i = 1,) (with j = 0,1, ) and
i = 3 (j = 1,2,-..). Since F k + . = F . ^ F ^ + F.__ 2 F k+l (see (1.10)), we obtain
from [9, (6b)], using (6.1) and (62) above, that
j
(6 4)

'

iJ

i-4
f c + 1)JF

M F

k+i? " E

k=o

i-3-k

(J = 1. 2, ) .

k=o

SFoting (6.1), (6.2), and (6.4), we are tempted to define

o, J

I>FkG?
k=o

= 0,1,

1967]

AND IDENTITIES FOR FIBONACCI NUMBERS

39

It should b e noted that (6.1) and (6.2) a r e not uniquely defined,,

In t h e

notation of [9, (8)], our (1.2) (with r = 0 and H, = F, ) can be w r i t t e n a s


(6.5)

S ( m s n - 1) = F P ( m , n ) + F
n

where (using

, P 0 ( m , n ) + C(m)
n~~ i

9, (2b), (3b) )
C(m) = ( - l ) m + 1 M

(6.6)

(m = 0 , l , - " ) .
2, m

T h u s , from (1.2), we obtain


_j

(6 7)

39J

= ( i)j

- 2]("1)k(k!)F^Gf

(J = 0,1,

"") ?

(6.8)

1 J

M23J = ("" ) ^]<-" 1 ) k ( k | ) F 2 k + i G f

<j = 0,1,---) .

k=o
Since M . = M . + M, . for Jj = 1, 2 , e " , w e obtain from (6.7) and (6.8) that
3SJ

2,3

1,3

(6.9)

Muj

1 3

= <"" ) ^<-" 1 ) k < kl


k=o

Since F 2 k + i - 1 = F i 1 F 2 k 4 - 1
u s i n g (6.8) and (6.9), that

+ F

)F

i-2:F3k

2k G j ? :

(see

0 = i'2'"')

(la0))s

w e

M.^ = <-l)J<-l> k W >F2k+i-iGf " J >

(6.10)

k=o

obtain

f r o m

t9*

(6b)

'^i-S-k

k=o

(J = 1, 2, )
F r o m (6.4) and (6.10), we conclude that
j
j

j
k

(6.11) (-l) ^(-l) (k!)F 2k+i _ l GJ


k=o

= Ek'Fk+iG^
k=o

(j = 1, 2, ; i = 0,1, ).

40

ON SUMMATION FORMULAS

[ Feb-

It should be noted that [9, (7c)] was obtained from [9, (6a)], using [9,
(7a)]. Since 1.9, (7c)] is a linear difference equation of second order in i, its
solution is
i-3

(6.12) P.(m,n) = F . ^ ( m , n ) ,+ F . ^ P ^ m . n ) - ^ ( n - k ) m F . ^ 1 _ k
k=

ft

= 3.4..

Using (6.12) and (1.10), [9, (8)] can be simplified to


(6.13)

P (m,n) + (-l) m + 1 M 0

S(m,n - h) = F P (m, n) + F
n

n+i

2,m

k=o

Since P 3 (m, n) = (-1)

P 3 (m,-n) [9, (9)] can be simplified (using [ 9,

(6a). (7c)]) to
n
m
TYi
(6.14) ^ ( n - k + l ) mm
F u = M<
_ FF _ , A +
+ M
M n _ FF _ +
" n
i.m n+i
2,m n+2
k=i

+ (-l) m + 1 (P 2 (m,-n) + Pidn.-n))

Since (see 19, (11)]) P.(m,n) = (-1) Q(m,-n + i - l ) ,

(m = 1, 2, - . ) .
where Q(m,n)

are the Weinshenk polynomials in n of degree m (see reference [8] cited in


[9]), it follows that
m
(6.15)

Q(m.n) = ( - l ^ P ^ m , n )

Thus (6.15), where M

J ] ( ^ )

^ f

is defined by (6.1), affords a closed form for the co-

efficients of Q(m,n)0 From (6.12), with n replaced by -n,


following recursion relation for the Weinshenk polynomials:

we obtain the

1967 ]
(6.16)

AND IDENTITIES FOR FIBONACCI NUMBERS


Q(m,n + i - l )

= F ^ Q t m , ! ! + 1) + F.

41

Q(m,n)

1-3

J2{n+k)mFi-i-k

(i = 3 4 8

> > ">

k=o

In j.9, (7a)] there is defined


m
(6.17)

P.(m,n) = ^ T (-l) j f\ M . ^ ^
j=o
V /

(m = 0,1,<

If we apply the well-known inverse pair relations,

k=o

k=o

^ '

to (6.17), we obtain as its inverse


m

(6.19)

M. jm - ] T V l ) 3 ( ^ ) P.O.n)!!111"5

(m = 0,1, ) .

Since P.(j,n) = (-l) J Q(j,-n + i - 1), we obtain from (6.19)


m

(6.20)

Q(J5

MUm = Y^lf)

~n + * " 1)nm"j

5=0
From (1.19), we obtain for n = 0, recalling (6.9),
m
(6.21)

(-Dm^)F2m

(-D^i.j

j=l

(m = 1, 2, )

42

ON SUMMATION FORMULAS

F r o m (1.20), we obtain for n = 0,

[Feb,

r e c a l l i n g (6.8),
m

(622)

(^l) (ni!)F2m+1 = ] T ( - l ^ M ^

(m = 0 , 1 , ) .

3=0
F r o m (2.35), we obtain, r e c a l l i n g (6.9),
m
(6.23)

ml F

_,_, = 7 S j M, .
m+l
L^J
m 1,3

(m = 1, 2, ) .

j=i
F r o m (2.36), we obtain, r e c a l l i n g (6.8),
(6.24)

mSF

If we set

= V S j Mo .
m+2
L^j m 2,]
j=o
JO

(m =

b = 2 in (4.3), then U

(4,20), set P(k) = k

= (~l) n is a solution of (4.3).

so that a

(4.20), with b = 2 and r = 0,

0,1,-0.

= 1, a. - 0, j = 0, l s - , m - 1.
m
j
gives a closed f o r m for

In

Thus,

n-l
vk, m
^(~l)kkm

k=o

ACKNOWLEDGEMENT
T h i s p a p e r , consisting of the f i r s t five s e c t i o n s , w a s submitted to t h i s
Q u a r t e r l y on S e p t e m b e r 2 1 , 1964, after the author reviewed t h e p a p e r by Alfred
[l].

Section6 of t h i s p a p e r w a s w r i t t e n , in e s s e n c e , after October, 1966, after

having r e a d the p a p e r by Ledin [ 9 ] . I wish to thank Dr. Hoggatt for the o p p o r tunity of r e a d i n g the p a p e r s [ l ] and [9 ] before t h e i r publication. As a r e s u l t ,
m y p r e s e n t p a p e r is m o r e complete,

1967 ]

AND IDENTITIES FOR FIBONACCI NUMBERS

43

REFERENCES
n
1. Brother U. Alfred, "Summation of T ^ F ,
k=i

k+r

Finite Difference Approach, "

Fibonacci Quarterly, Vol. 5 (1967), pp 91-98.


2.

C. Jordan, Calculus of Finite Differences, Chelsea, New York, 1960.

3. R. Go Buschman, "Fibonacci Numbers, Chebyshev Polynomials, Generalizations, and Difference Equations," Fibonacci Quarterly, Vol. 1, (1963),
No. 4, pp. 1-7.
4. R. I. Reichman, "A Summation Formula Involving Fibonacci Numbers, n
Scripta Math. Vol. 20, March-June, 1954, pp. 111-112.
5 K. Subba Rao, "Some Summation Formulae Involving Fibonacci Numbers, "
Scripta Matho, Vol. 21, June-Sept., 1955, pp. 214-217.
n
kp
k=o
Soco , 15(1964), pp. 642-6470

6. D. Zeitlin, "On the Sums

and

n
,
(-1) kP, " Proc. Amer, Math.
k=o

7. D. Zeitlin, "On Summation Formulas for Fibonacci and Lucas Numbers, "
Fibonacci Quarterly, Vol. 2 (1964), pp. 105-107.
8. L J. Schwatt, An Introduction to the Operations with Series,

Chelsea,

New York, 1962.


9. G. Ledin, "On A Certain Kind of Fibonacci Sums, " Fibonacci Quarterly,
VoL 59 No. 1, pp. 45-58.
* *

THE Q MATRIX AS A COUNTEREXAMPLE IN GROUP THEORY


D. A . L I N D , University of V i r g i n i a , Charlottesville, V a .

If g is an element of a group G, then o(g), the order of g, is defined


to be the number of distinct elements of G in the set j e , g , g , 8 j 5 where
e is the identity of G. This is equivalent to defining o(g) to be the number
of elements in the cyclic subgroup of G generated by g. It is an easy consequence that the order of g equals the least positive integer n such that g n =
e. If no such integer exists, g is said to be of infinite order*
In an abelian group H (i. e. , ab = ba for all a,b 6 H) it is easy to
show that the product of two elements of finite order must again be of finite
order.

Indeed, if o(a) = m, o(b) = n for some a,b H, then


TTL

T\

(ab)

TO

= e, so o(ab) < mn. However, this does not necessarily


hold in general, as shown in the following counterexample involving the Q
matrix*
Let G be the multiplicative group of all nonsingular 2x2 matrices, and
let

R =

["o i ] '

S=

["i ~]

be elements of G. One can check that K 2 = S3 = I,


that R and S are of finite order,,

the identity matrix, so

But

RS = [I J] = Q .
the Q matrix. Now Basin and Hoggatt [l] have shown that
(RS)n

= Qn =

"n+i

^ I
"n-i

for any n > 08 Thus RS has infinite order,


(See page 80 for reference.)
44

ON A CERTAIN KIND OF FIBONACCI SLIMS*


GEORGE LEDIN, J R . , Institute of Chemical Biology, University of San Francisco, San Francisco, C a l i f .

INTRODUCTION
The sum
n

S(m,n) = J ] k m F k
k=i

(where F, is the k
ues of me

Fibonacci number) has been studied for particular val-

The cases m = 0 and m = 1 are well known [1,2]

The case

m = 3 was proposed as a problem [3] by Brother Ue Alfred of St. Mary ! s


College, California; this problem was later solved [4] by means of translational operator techniques and linear recurrence relations [5], This

method

of solution [4] can be generalized for arbitrary positive integral values of m,


but it usually will involve the time-consuming, error-inviting procedure of
solving 2m + 2 simultaneous equations in 2m + 2 variables, which is already
a complicated task for m = 3.
The method outlined in this paper is much more elementary, and the
work required in finding a particular sum is reduced to several simple integrations. The procedure discussed below not only facilitates the computation
of these sums, but it is also a useful tool in the solution of other problems^
such as the problem of Fibonacci If centroids fr proposed by the author [6], c e r tain aspects of Fibonacci convolutions, and the like.
THEORY
Consider the sum
n
W

2 k m F k = S(m,n) = F n + 1 P 2 (m,n) + F ^ d n . n ) + C(m)


k=i

*This paper was originally presented at the Fibonacci Association Meeting of


21 May 1966.
45

46

ON A CERTAIN KIND OF FIBONACCI SUMS

where F, denotes the k

[ Feb.

Fibonacci number (F 0 = 0, F t = 1, F^+2 = F k+t

+ Fk), P^m.n) and P 2 (m,n) are polynomials in n of degree m, and C(m)


is a constant depending only on the degree m
Thus we can write
(2a)

P^rn^n) = a m n m + an^n 3 1 1 " 1 + + aln + a,

(2b)

P 2 (m,n) = b m n m + b m 1 n m

+ . . . + b4n + b 0

Theorem 1.
C(m) = -b 0
Proof.
Take S(m, 0) = F ^ m , * ) ) + FoP^m.O) + C(m)
from (1))
0 = P 2 (m,0) + C(m) but P 2 (m,0) = b0 from (2b)) .
Inspection of the first few values of m (see Table I) leads us to the following determination of the polynomials (2a) and (2b).
m

P^m.n) = ( - l ) J ( ^ M l 7 J n H

(3a)

ILL

P2(m,n) ^ ( - ^ ( r ) ^ , ^

(3b)

where I . I are the binomial coefficients, and M l s j and M 2 j are certain


numbers,
s, the law of formation of which is yet to be determined (refer to Table II).
Theorem 2.
(4a)

n
P^m + l,n) = (m + 1) J P^m.njdx + a{
o

(4b)

P 2 (m + l,n) = (m + 1) J

n
P 2 (m,x)dx + bj

ON A CERTAIN KIND OF FIBONACCI SUMS

47

Table I
LIST O F FIBONACCI SUMS OF THE TYPE
S m

( > ) ^ Z k ^ ^ k

n+i

2(m'n)

F ^ O ^ n ) + C(m)

S(0,n) = F n + 1 ( l ) + F n ( l ) - 1

S ( l , n ) = F n + l ( n - 2) + F n ( n - 1) + 2

S(2 9 n) = F , (n2 - 4n + 8) + F (n2 - 2n + 5) - 8

S(3,n) = F n + i ( n 3 - 6n 2 + 24n - 50) + F n (n 3 - 3n 2 + 15n - 31) + 50

S(4 $ n) = F

S(5sn) = F n+l (n 5 - 10n4 + 80n3 - 500n2 + 2080n - 4322) +

(n4 - 8n3 + 48n 2 - 200n + 416) + F ( n 4 - 4 n 3 +30n 2 - 124n +

+ 257) - 416
+ F (n5 - 5n4 + 50n3 - 310n2 + I285n - 2671) + 4322
6

8(6,11) = F

(n6 - 12n5 + 120n4 - 1000n3 + 6240n2 - 25932n + 53888) +

+ F (n6 - 6n5 + 75n4 - 620n3 + 3855n2 - 16026n + 33305) - 53888


7

S(7,n) = Fn+1(n7 - 14n6 + 168n5 - 1750n4 + I4560n3 - 90762n2 +


+ 3772l6n - 783890) + F (n7 - 7n6 + 105n5 - 1085n4 +
+ 8995n3 - 56091n2 + 233135n - 484471) + 783890

S(8sn) = F n+1 (n 8 - 16nT + 224n6 - 2800n5 + 29120n4 - 242032n3 +


+ 1508864n2 - 627112On + 13031936) + F (n8 - 8n7 + 140n6 n
- 1736n5 + 17990n4 - 149576n3 + 932540n2 - 3875768n +

10

+ 8054177) - 13031936
S(99n) = F ,,(n9 - 18n8 + 288n7 - 4200n6 + 52416n5 - 544572n4 +
n+i
3
+ 4526592n
- 28220040n2 + 117287424n - 243733442) +
+ F (n9 - 9n8 + 180n? - 2604n6 + 32382n5 - 336546n4 +
n
+ 2797620n3 - 17440956n2 + 72487593n - 150635551)
+ 243733442
S(10,n) = F
(n10 - 20n9 + 360n8 - 6000n7 + 87360n6 - 1089144n5 +
n+i
+ 11316480n4-- 94066800n3 + 586487120n2 - 2437334420n +
+ 5064892768) + F (n10 - 10n9+ 225n8 - 3720n7 + 53970n6 - 673092n5 + 6994050n4 - 58136520n3 + 362437965n2 - 1506355510n + 3130287705) ~ 5064892768

48

LFeb.

ON A CERTAIN KIND OF FIBONACCI SUMS


Table II
LIST OF THE M 1$ j AND M 2 j j

NUMBERS

M i.5

M< 2 . 3

31

50

257

416

2671

4322

33305

53888

484471

783890

8.

8054177

13031936

150635551

243733442

10

3130287705

5064892768

(5a)

aj = 1 - (m + l)f

( P ^ m . x ) + P 2 (m,x))dx

(5b)

bj = 1 - (m + 1) J (Pi(m,x) + 2P 2 (m,x))dx
o
Proofo
P r o v e (4a) f i r s t .

Using (3a) we have

xxx

(m + 1) { P i ( m , x)dx = (m +

iX

/
m

Cm + 1) X V D X j d) ^
H

QX

n
C

= E (-DV 3 ( m + 1) n+H
j=

= Pi(m + l,n) - aj,

1967]

ON A CERTAIN KIND OF FIBONACCI SUMS

49

(aj is determined for j = m + 1, a value which is missing from the summation sign.) A similar proof establishes (4b).
Now,
l

aj = PjCm + 1, 0) = Pi(m + 1,1) - (m + 1) J Pi(m 9 x)dx


o
and
l

bj = P 2 (m + 1,0) = P 2 (m + 1,1) - (m + l ) / p 2 ( m , x ) d x
o
and since S(m + 1,1) = 1 = P 2 (m + 1,1) + Pj(m + 1,1) + C(m + 1)

(C(m + 1) =

-bj by Theorem 1) then


l

1 = (m + 1) J Pi(m,x)dx + aj + (m + 1) J P 2 (m,x)dx
o
o
and the value of a* follows,, A similar manipulation yields the required value
of bj .
Corollary 1
dPjdn + l,n)
dn

dP 2 (m + l,n)
= (m + D P i1dVn , ! ! ) ;

= m(m + l)P 2 (m,n)


'
'
^'^'
A r
dn

Corollary 2
d r P!(m,n)

d r P 2 (m,n)

= m(m - l) (m - r + l)Pj(m - r, n);


dn

= m(m - 1)
dn
(m - r + l)P 2 (m - r, n)

Corollary 3
P 2 (m, 1) = a0

(refer to (2a, 2b)).

Example 1
Problem.

Obtain the sum

n
\ J kF,
k=i

50

ON A CERTAIN KIND OF FIBONACCI SUMS

[Feb.

Solution. We know
n

E F,k

= F ^ + F - 1
n+i
n

(m = 0) .

k=i
So the polynomials are Pt(0fn) = 1, P2(0sn) = 1. Now, applying Theorem 29
Pi(l,n) = J l d x + aj = n + aj
o

and

P 2 (l,n) = f
o

Idx + b^ = n + bj

aj = 1 - J* (1 + l)dx = 1 - 2 = 4

and

Thus, the required sum is equal to F

bj = 1 - / (1 + 2)dx = 1 - 3 = -2

(n - 2) + F (n - 1) + 2,

Example 2
Problem.

Obtain the sum


n
k=i

Solution, From Example 1, we know

EkFk
k=i

= F

n+i{n"2)

+ F

So the polynomials are Pi(l,n) = n - 1,

n(n"

1) + 2

P 2 (l s n) = n - 2

Nowf applying

Theorem 2

Pi(2,n) = 2 J (x - l)dx + aj = n2 - 2n + aj and P2(2sn) = 2 J (x- 2)dxH-bg = n 2 ~4n+bj

1967 ]

ON A CERTAIN KIND OF FIBONACCI SUMS


1

51

J = l - 2 j ( x - l + x - 2)dx = 1 - 2 J (2x - 3)dx = 1 - 2(1 - 3) = 1 + 4 = 5

bJ = l - 2 / ( x - l + 2 x - 4)dx = 1 - 2 J (3x - 5)dx = 1 - (3 - 10) = 1 + 7 =

Thus, the required sum is equal to F

(n2 - 4n + 8) + F (n2 - 2n + 5) - 8

Theorem 3.
If u, are the "generalized" Fibonacci numbers (i e, numbers obeying
the Fibonacci recurrence relation, but with different initial conditions) with
the properties u k + 2 = \

+ 1

u0 = q, uA = p, [ 7 ] , then

E kmu

= u n + 1 P 2 (m, n) + u n P i (m, n) + K(m) ,

k=i

where P 2 and Pi are polynomials defined as above (3a9 3b) and K(m) =
-(pb 0 + qa0 ).
In Theorem 3 we have stated a simple and useful result.
this theorem is trivial, since u, = pF^ + qF,

The proof of

[ 7 ] . Two particular cases

are most interesting. The Fibonacci case (p = 19 q = 0) has been discussed


above; the Lucas case (p = 1, q = 2) is also quite simple (refer to Table HI).
At this stage it seems clear that a study of the polynomials Pi(m 9 n) and
P 2 (m,n) and of the numbers M y and M y pose by themselves an interesting problem. The intuitive bounds
M y + 1 > 2(j + l ) M y

M y + 1 > 2(j + l ) M y

(j > 1)

hold for all cases shown on Table II and can be proven by total induction using
the formulas developed for aj and bj

A very curious relationship exists

between these numbers; this relationship, and the fact that these numbers are
members of a whole class of numbers M y can be appreciated effectively in
Table IV Horizontal addition of two consecutive M y numbers is the basic

52

[Feb.

ON A CERTAIN KIND OF FIBONACCI SUMS


T a b l e IE
LIST OF LUCAS SUMS OF THE T Y P E
T(m,n) = E k = 1 k m L k

= L ^ P ^ n )

m = 0

T(0,n) = L n + 1 ( l ) + L n ( l ) - 3

m -

T ( l , n) = L n + l ( n - 2) + L n <n - 1) 4- 4

+ L^fr^n)

+ K(m)

m = 2

T ( 2 , n ) = L n + i ( n 2 - 4n + 8) + L n (n 2 - 2n + 5) - 18

m = 3
m = 4

T(3 9 n) = L ^ ( n 3 - 6n 2 + 24n - 50) + L (n3 - 3n 2 + 15n - 31) + 112


n+i
n
T(4, n) = L
fn4 - 8n3 + 48n 2 - 200n + 416) + L (n4 - 4n 3 + 30n 2 - 124n
n+l
n
+ 257) - 930

m = 5

T(5 s n) = L n + 1 ( n 5 - 10n 4 + 80n 3 - 500n 2 + 2080n - 4322) +


+ L (n5 - 5n 4 + 50n 3 - 310n 2 + 1285n - 2671) + 9664
n

p r i n c i p l e in the c o n s t r u c t i o n of T a b l e IV; t h e r e s u l t s of s u c c e s s i v e h o r i z o n t a l
additions can be followed with t h e aid of t h e b r o k e n l i n e s . T h e following i l l u s t r a t i o n should clarify t h e p r o c e s s ;

1
\

^^
\

_
\

-31*"

^ \
M+5=6 -^ _ _
5+31=3
1
~ ~ ^ 6+36=42

1
2<L

2+6=8
_ _

\
^.8+42=50
50-

^8-

T h e s e z i g - z a g r e l a t i o n s h i p s imply t h e s e c o n d - o r d e r l i n e a r difference equation


(6a)

= M

+ M

(i = 3, 4, 5 , - - ;

- (i - 3) 3

j = 0, 1, 2,--- )

t h e solution of which i s shown in Eq. (6b)

1967 ]

53

ON A CERTAIN KIND OF FIBONACCI SUMS


Table IV
INTERDEPENDENCE CHART FOR THE M. . NUMBERS

-31-

-257-

-2671

/
\
288-__
2928
- 42 - _ _ _ "^ - - 3 2 4 - _ _
-3216
/
\
^ - - 3 6 6 - _ _
-3540
7
X
/
\
^
^
3 9 0 6
/
x

6 ^ _

/ \

36 -

\ /
\

58-^_
68-_
/ \

^ - \

1 3 -/

-416-

-4322

466-__
- - 524- _ .
- - 592- _

\
/

io^.
/ \

-50-

\
\

\
/
- 8 1

4738
' - -5204
"^-5728
----6320
-6993

6 73*

1 6 ^ _
/ \
/
\
/
\
/
\

\ 9 4\/x - _ _ _
" - -110 - _ _
/ \
/
\

X
754-~^__
7666^
^ ~~ - 8 4 8 - _ _ __ - - 8 4 2 0
"--958--__' ^--9268
/
\
~"^-10226

'

\2 0

1/

130-

1088N

'

-11314
//

24^__
150^___
1218-__
12402^
\
^ - - 1 7 4 - ^ _ _ _ ^--1368-^ __^~^-13620
/
\
/
\
^--1542-_^"~~"^-14988
X
/
/
\
/
\
^-16530
\
\
/
\
/
\
/
/
\ /
\
/
\
/
\
29203-
1745^
/

[/

X /

34-^-___
232-___
1948-___
20020
/ \
^-266-__
^^-2180 _ _ _
"~~-21968
/
\
/
\
~^^--2446___ ^--24148
/
\
/
\ x
'
\
^--26594
/
\
/
/
\ //
X
\
/
402752-/
-306-

18275

\ 29346

N 3. 46 /- _ _ _
N 3M05 /8 X _ /
32098
^
3
9
2
_
_
_
^
3
4
0
4
^-35156
/
/ \
^
^--^3796- _
--38560
/
X
/
/
/
~^-42356x
x
/
\
/
\ N
/
\
/
\
| /
N
. 445

- X 4241^
46597
?7
5 3
4 6

/ ^
\
\
\

54

ON A CERTAIN KIND OF FIBONACCI SUMS

[ Feb.

i-4
(6b)

M. . = F . ]VL . + F . M, . - V * (k + 1)^ F . Q ,
i,j
l-i 2, j
i-2i,]
L^
i-3-k
k=o

w h e r e F . r e p r e s e n t s the i '

Fibonacci numbero

T h e interdependence of the fundamental set of n u m b e r s M ^ j and

M2jj

is noted f r o m the f o r m u l a s
3

(60

MlfJ = E ( - 1 ) h ( h )
h=o

and

2,J-h

2,j

'

E
h=o

(h)
V

^"h

'

The interdependence of the c o m p l e t e set of n u m b e r s M. . is evidenced with the


formula1:

<6d)

M ^ . = (i - 1)1

(2j"h - 1) ( i ) M.jh

h=o

with j > 0, M. = 1,
J
*
i,o

M. = i > 1.
1,1

David Zeitlin, in a p a p e r to b e published in the F i b o n a c c i Q u a r t e r l y , 2 h a s


shown that the following r e l a t i o n s h i p holds:
j

M. . = V h ! $ h F ^ .

(6e)

i.J

^
h=o

h+i

w h e r e $ . a r e t h e Stirling n u m b e r s of t h e second kind,


T h e polynomials

V\ and P 2 a r e , s i m i l a r l y ,

s p e c i a l c a s e s of a m o r e

g e n e r a l c a s e of polynomials.

The author i s indebted to D r . V e r n e r E Hoggatt, J r . for pointing out t h i s r e lationship through p e r s o n a l c o r r e s p o n d e n c e .


2
The author acknowledges the r e f e r e e for t h i s i n t e r e s t i n g r e m a r k .

1967 ]

ON A CERTAIN KIND OF FIBONACCI SUMS

m
P.(m f n) = Y (~1)3M. . H

(7a)

nm~j

isj I ] I

L^

55

X !

j=0

which are interrelated in the following ways?


(7b)

P. + h (m s n) = P . f r ^ n - h )

(7c)

P.(m3n) = P. .(m,n) + P.

(msn) - (n + 3 - i) m

(i = 3, 4, 5, )
These properties (7) enable us to obtain the following formula, thus generalize
(1):
(8)

S ( m 9 n - h ) = F n ^ h + 1 P g ^(m 9 n)+ Fn_hP1_i_h(m5n) + C(m)


We have investigated sums of the form
Ft + 2 m F 2 + 3 m F 3 + + (n - l ) m F
l

+ nmF

n _ 1

and it seems quite natural* that we apply our results to the "convolution type1
sums of the form
n m F i + (n - l ) m F 2 + (n - 2 ) m F 3 + . . . + 2 m F n - 1 + F n
Theorem 40

(9)

(n

"

k + 1 ) m

R(m n)

'

k=i

* Mathematicians 1 beloved excuse*

s5mFn+i

+ M

2 3 m F n " P 3* (m > n)

56

ON A CERTAIN KIND OF FIBONACCI SUMS

where

and M
o, m

are particular

c a s e s of the

<i|,m

[ Feb.

M. . n u m b e r s
i,j

(see

T a b l e IV) and Pj^m, n) (the "conjugate" of the polynomial P 3 (m, n)) is defined
a s follows
in

(10)

P*(m,n) = ^ M 3 j j H

A l i s t of t h e s e "convolution-type" s u m s i s provided in T a b l e V.
Table V
n

LIST OF SUMS OF THE T Y P E

2 >
k=i
m = 0
m = 1
m = 2
m = 3
m = 4
m = 5

V 1 ^ xF ^
" k + 1 ) ^k = xv
Rvu^
( m , niv) = iM
3jin n + 1

V i ? _ x F - P 3 ( m , n )
+ XM
2,m n

R(0,n) = F ^ + F - 1
n+l
n
R(l,n) = 3F _,_ + 2F - (n + 3)
n+l
n
R(2,n) = 13F ^ + 8F - (n2 + 6n + 13)
n+i
n
R(3,n) = 81F ^ + 50F - (n3 + 9n 2 + 39n + 81)
n+l
n
R(4,n) = 673F ^ + 416F - (n4 + 12n 3 + 78n 2 + 324n + 673)
n+l
n
R(5,n) = 6993F , A + 4322F - (n5 + 15n4 + 130n3 + 810n2 + 3365n + 6993)
n+l
n

If Q(m,n) a r e the Weinshenk polynomials in n of d e g r e e m


(11)

Pt(m,n) = Q(m,n + i - l )

and

P.(m,n)

[ 8 j , then

= ( - l ) m Q ( m , - n + i - 1)

T h e above r e l a t i o n s h i p s (11) follow from the fact that P*(m, n) = (-1)

P.(m,-n).

T h e constant t e r m i s then C(m) = P i ( m , l ) = Q(m, 1), and t h e o r i g i n a l s u m (1)


can b e f u r t h e r w r i t t e n a s follows:

(12)

S(m,n) = ( - l ) m J F n + 1 Q ( m 5 - n + 1) + F n Q ( m , - n ) - Q ( m , l ) }

T h e t h e o r e t i c a l i n t e r e s t that t h e s e s u m s a r o u s e i s beyond doubt the p r i m a r y motive for t h e i r s c r u t i n y .

Weinshenk [8 ] has applied s o m e of

these

1967 ]

ON A CERTAIN KIND OF FIBONACCI SUMS

57

r e s u l t s to a p r o b l e m of reflection of light. T h e p r o b l e m of c e n t r o i d s [ 6 ] can be


dealt in a m o r e g e n e r a l m a n n e r with the aid of an auxiliary function defined by

(13)

G(r,s,n) =

hi p a r t i c u l a r ,

G ( l , 0 5 n) = G

h a s t h e following limiting b e h a v i o r :

G
-S-JL = l i m (G ^ - G ) = 1
G
^
n+i
n
n>oo
n
n> oo
lim

The p r o b l e m s investigated in t h i s p a p e r a r e f a r f r o m being completely


solved. Although we could have g e n e r a l i z e d the s u b s c r i p t s in all our s u m s [ 9 ] ,
we p u r p o s e l y avoided
been a n s w e r e d .
1.

this* However, s o m e questions of i m p o r t a n c e have not

Some of t h e s e questions a r e :

Could t h e t h e o r y of S(m,n) be extended to negative m ?

(All we need

to study is m = - 1 , s i n c e t h e r e s t of t h e s u m s can be obtained with t h e aid of


t h e a l g o r i t h m s developed in t h i s p a p e r ; notice that

^2P.(m,n)
P.(-l,n)
i

2.
m?

lim
^
m*>o

dndni

Could t h e t h e o r y of S(m, n) be extended to r a t i o n a l (and to real) [ l o ]

If t h i s is p o s s i b l e , what can be said about complex


3.

m?

What is the p o s s i b i l i t y of studying s u m s of the type

S(r,s,n) = krFjJ
k=i

with the aid of s t a n d a r d t e c h n i q u e s ?

58

ON A CERTAIN KIND OF FIBONACCI SUMS

F e b . 1967

ACKNOWLEDGEMENTS
The author w i s h e s to e x p r e s s h i s g r a t i t i d e to Dr. V e r n e r E. Hoggatt, J r .
and the r e f e r e e , who had v e r y useful and c o n s t r u c t i v e c o m m e n t s on s e v e r a l
a s p e c t s of t h i s p a p e r .

REFERENCES
1.

S. L. Basin and V. E, Hoggatt, J r . ,

'A P r i m e r on the Fibonacci Sequence,

P a r t I , " T h e Fibonacci Q u a r t e r l y , Vol. h No. 1, (1963) p . 66.


2.

S. L. Basin and V. E. Hoggatt, J r ,

ff

A P r i m e r on the Fibonacci Sequence,

P a r t n , " T h e Fibonacci Q u a r t e r l y , Vol. 1, No. 2, (1963) p . 67.


3.

P r o b l e m H-17 of "Advanced P r o b l e m s and Solutions," T h e Fibonacci Q u a r t e r l y , Vol. 1, No. 2 (1963) p 0 55.

4.

Solution to P r o b l e m H-17, The Fibonacci Q u a r t e r l y , Vol. 2, No. 1, (1964)


p . 51.

5.

J a m e s A. J e s k e , " L i n e a r R e c u r r e n c e R e l a t i o n s " ( P a r t I), The Fibonacci


Q u a r t e r l y , Vol. 1, No. 2, (1963) p. 69.

6a

P r o b l e m H-57 of "Advanced P r o b l e m s and Solutions," The Fibonacci Q u a r t e r l y , Vol. 3 , No. 1, (1965) p . 45.

7.

P r o b l e m H-44 of "Advanced P r o b l e m s and Solutions," The Fibonacci Q u a r t e r l y , Vol. 2, No. 3, (1964) p . 205.

8.

Ronald Weinshenk, "Convolutions and Difference Equations A s s o c i a t e d with


with t h e N-Reflections

of Light in Two G l a s s P l a t e s , " ( M a s t e r ' s T h e s i s ) ,

San J o s e State College, J u n e 1965.


9.

K e n S i l e r , " F i b o n a c c i Summations, " The Fibonacci Q u a r t e r l y , Vol. 1, No.


3, (1963) p . 67.

10.

E r i c Halsey, " T h e F i b o n a c c i Number F , w h e r e u is not an Integer, " The


F i b o n a c c i Q u a r t e r l y , Vol. 3, No. 2, (1965) p 8 147.

*****

i LATTICE POINT SOLUTION OF THE GENERALIZED PROBLEM OF TERQUEi


. AND AN EXTENSION OF FIBONACCI NUMBERS .
C . A . CHURCH, J r . and H. W . G O U L D , W . Virginia University, Morgantown, W . V a .

In this paper we give a simple lattice point solution to a generalized permutation problem of Terquem and develop some elementary results for the
extended Fibonacci numbers associated with the permutation problems
The classical permutation problem of Terquem [12] has been stated by
Riordan [10, p 17, ex 15] in the following manner,.

Consider combinations

of n numbered things in natural (rising) order, with f (n,r) the number of r combinations with odd elements in odd position and even elements in even positions, or, what is equivalent, with f (n,r) the number of combinations with an
equal number of odd and even elements for r even and with the number of odd
elements one greater than the number of even for r odd.
It is easy to show that f(n,r) = f (n - 1, r - 1) + f(n - 2, r), with f(n, 0)
= 1, and explicitly

(1)

f(n,r) =

Moreover,
n
(2)

f (n) = ^T f (n, r) = f (n - 1) + f (n - 2)
r=o

so that f(n) is an ordinary Fibonacci number with f(0) = 1 and f(l) = 2


A detailed discussion of Terquem f s problem is given by Netto [8, pp. 8487] and Thoralf Skolem [8, pp. 313-314] has appended notes on an extension of
the problem in which the even and odd question is replaced by the more general
question of what happens when one uses a modulus m to determine the position
of an element in the permutation.
"^Research supported by National Science Foundation Grant GP-482.
59

60

LATTICE POINT SOLUTION OF THE GENERALIZED


M o r e p r e c i s e l y , for a modulus m > 2,

stated a s follows.

[Feb.

Skolem T s g e n e r a l i z a t i o n m a y b e

F r o m among the f i r s t n n a t u r a l n u m b e r s let f(n,r;m)

de-

note the n u m b e r of combinations in n a t u r a l o r d e r of r of t h e s e n u m b e r s such


that the j
(3)

element in t h e combination i s congruent to j modulo m. That i s ,

f(n,r;m) = N < a 1 a 2 o e a r :

1 < a* < a 2 < * " < a r < n,

a. = j (mod m)>

1< B,t< a 2 < < a r < n


a. = j (mod m)
C o n s i d e r the a r r a y in F i g . 1, w h e r e the l a s t e n t r y is

r + km,

with

k =
m
s i n c e r + k m < n i m p l i e s that t h e l a r g e s t i n t e g r a l v a l u e of k cannot exceed
(n - r ) / m 0

T h i s a r r a y contains t h o s e , and only t h o s e , e l e m e n t s from among

1, 2, , n which m a y a p p e a r in a combination,,
s i s t s of all t h o s e e l e m e n t s < n
m a y a p p e a r in the j

column con-

in t h e s a m e congruence c l a s s (mod m) which

position.

(0,0)
1

#-$* X

1 +m l + 2m

i-

That i s , t h e j

1 + km

i.

2 + m -

-3 + m -

-2 + 2m

-3 + 2 m

-2 + k m

i.

3 + km

r +m
2m

-i.

-r + km #
B = (r,s)

Fig 0 1

F r o m t h e l a t t i c e appended to the a r r a y In Fig 1, we can s y s t e m a t i c a l l y


w r i t e out t h e d e s i r e d combinations, and evaluate f(n, r;m)
To get the d e s i r e d r e s u l t , let " a path from A to BfT m e a n a path along
t h e v e r t i c a l and h o r i z o n t a l s e g m e n t s of t h e l a t t i c e , always moving downward o r
from left to right (we t a k e the p o s i t i v e x - a x i s to the right, t h e p o s i t i v e y - a x i s

1967]

PROBLEM OF TERQUEM AND AN EXTENSION


61
OF FIBONACCI NUMBERS
downward, thus agreeing with the informal way of writing down the permutations)*
Each such path will generate a combination of the desired type, and conversely,
as follows? Starting at A each horizontal step picks up an entry and vertical
steps line up entries 0

Now, It is well known how many lattice paths there a r e

from a = (0,0) to B = (r,s). MacMahon [7, VoL I, p 167] shows that this
number is precisely

(
hi our case s = [(n - r ) / m ] c

Thus we have at once that

r +

f (n, r;m)

(4)

(m - l)r"

m\[-

HI

as found by Skolem0 Terquem ! s (1) follows when m = 2

To illustrate, we

consider some examples 0


Example 1. Let n = 12, r = 3, m = 40 Then the corresponding array
is

1r

2i

fiD

3 _ J ^--10*-

and the ten combinations are


1 2

1 2

11

1 6
1
6

7
11

11

10

5
5

6
10

11
11

10

11

and the particular combination 5, 10, 11 corresponds to the path indicated by


arrows. Informally, one writes out the combinations by paths from the left
column to the right column, moving horizontally and/or diagonally,,

The clue

to a systematic count is found by superimposing the rectangular grid*


Example 2 Let n = 12, r = 4, m = 30 Then the corresponding array
is

62

LATTTCE POINT SOLUTION OF THE GENERALIZED PROBLEM [Feb*


2

FH

1|

-5*-

11

*7
t

11

. Q , JL^^io^-

and the fifteen combinations a r e


1 2
1 2
1

1 2
1

3
3

4
7

1 2 9
1 5
6

10
7

4
4

5 6
5 6

7
10

10

10

10

10

8 9

10

10

10

8 9

10

and the combination 4, 5, 9, 10 corresponds to the path indicated by arrows,


It is felt that our proof shows atruism of mathematics: one may often find
a simpler proof by embedding a given problem (Terquem f s) in a more general
setting., The lattice point enumeration we used is well known, but may not be
apparent in the original problem because of its specialized form 0
The extended Fibonacci numbers, in analogy to (2), are now defined by

n
(5)

f(n) = f m (n) =

j*n + (m - l)r"
m

r=o satisfy the recurrence relation


and it is not difficult to verify that they
(6)

f m (n) - f m (n - 1) + f m (n - m)

For example, with m = 3 we have the sequence 1, 2, 3, 4, 6, 9,13,19, 28,


By well-known theorems in the theory of linear difference equations, if the distinct roots, of the equation
(7)

t m - t111"1 - i - o

a r e ti, t 2 , t m * then there exist constants C r such that

1967] O F TERQUEM AND AN EXTENSION OF FIBONACCI NUMBERS

63

HI

(8)

(n) = \ ^ C

ZmJ

tn
rr

This generalizes the familiar formulas


-o
a - b
F
= -~
,
n
a - b

,
L

= a

n , ,n
+ b

for the F i b o n a c c i - L u c a s n u m b e r s . T h e c o n s t a n t s C

,
*

m a y b e d e t e r m i n e d from

t h e s y s t e m of m l i n e a r equations in C :
m

(9)

7 J C r t 3 r = j + 1,-

for

j = 0, 1, 2, , m - 1 .

r=i
F o r example 9 when m = 3, an a p p r o x i m a t e solution of the equation (7) i s given
by

w h e r e i2 = ' - 1 .

t j = 1.4655 ,
t2 -

-0.23275 + 0 o 792551 ,

t 3 = -0 o 23275 - 0 o 792551 ,

Relations (5) through (9) a r e given by Skolem [ 8 , 313-314]*

When m = 3 t h e exact solution of (7) i s given by


ti = A + B + -|

(11)

t2 = J - A + i
1

where

A + B

A^B

A - B ./-o

64

LATTICE POINT SOLUTIONS OF THE GENERALIZED PROBLEM [ F e b .

A s a p a r t i a l check on t h e v a l u e s of t h e r o o t s , we note t h e following t h e o r e m


from t h e t h e o r y of equations.

Let

n (t - t.) = t m - t m

(12)

- z

Then

m l

(13)

lL ]
j=i

\b>

" m) ^ '

n> 15

k=o

where

/ i.\
(a b)

k '

/ a + bk \

= rrbk (

T h i s may b e c o m p a r e d with the well-known [ 2 , 3, 4, 5 ]

xa = ^ T Ak(a,b)zk

(14)

with z =

,
k=o

expansion

x - 1
b
x

which w a s actually found by L a g r a n g e in his g r e a t m e m o i r of 1770 (Vol, 24 of


P r o c . of t h e B e r l i n Academy of Sciences) and which l e a d s at once to the g e n e r a l
addition t h e o r e m d i s c u s s e d in [ 2 , 3, 4, 5 ]
See r e l a t i o n (20), this paper,,
m
ml
F o r the equation t - t
- z = 0,
roots t .

(15)

a s f i r s t noted by H. A. Rothe
we define t h e power s u m s of t h e

by

S(n) -

l)

1967] OF TERQUEM AND AN EXTENSION OF FIBONACCI NUMBERS


Since t . "
3

65

+ z = t . , we find that
3

m
{t*" 1 + z t ^ 1 1 1 J =

S(n - 1) + zS(n - m) =
j=i

t ^ / t ^

j=i

+ z\

- J ]

t^t*1,

j=i

so that S(n) itself a l s o s a t i s f i e s a F i b o n a c c i - t y p e r e c u r r e n c e


(16)

S(n) = S(n - 1) + zS(n - m) .

Using t h e v a l u e s z = 1, m = 3, t h e p r e v i o u s r o o t s (10) yield t h e a p p r o x i m a t e


v a l u e s (by log t a b l e s ) : S(l) - 1,
very nearlyc

S(2) = 0 e 9998, S(3) - 3,9995, and S(4) = 5

T h i s gives a p a r t i a l check on (10)o

In any event 5 we m a y c o n s i d e r t h e s e q u e n c e defined by (13), (15), (16) a s


a kind of extended Fibonacci sequence,,

(17)

S(n)

n - (m - l ) k (

Z^
k=o

hi p a r t i c u l a r ,

|- (m
k

l)k

n > 1 ,

s a t i s f i e s (16) j u s t a s (5) s a t i s f i e s (6). T h e r e a r e s i m i l a r i t i e s and c o n t r a s t s if


w e c o m p a r e (17) and (5)0

We a l s o c a l l attention to a n o t h e r such r e s u l t given

r e c e n t l y by J A9 Raab [Vj, who found that the sequence defined by

(18)

[r+ij
\-^

/ n - rk\

n- k(r+i)

satisfies

(19)

= ax

n - lJ

+ bx

n-r-i

bk

LATTICE POINT SOLUTIONS OF THE GENERALIZED PROBLEM [Feb

Formula (13) is substantially that given by Arthur Cayley [!]<,

The classical

Lagrange inversion formula for series is inherent in all these formulas,,

One

should also compare the Fibonacci-type relations here with the expansions
given in [5]

For m = 3, (17) gives the sequence 1,1, 4, 55 6,10,15, 21, 31,

We als.o call attention to the two well-known special cases


, ,

(n i k) r -2kzk

n+i
__ x
=

n+i
- y
x - y

k=o

and
V^ / n - k \

k n k

=r \ / -

k=o

n-2k-i k

n +, y n
x11

'

x + y

where x = 1 + Vz + 1, y = 1 -v/z + 1 F and L occur when z = 49


J
v
n
n
Relations (17) and (5) differ because the initial conditions differ., For
z = 1,

(17) satisfies precisely the same recurrence as (5)c

If the initial

values were the same then we would have found a formula for the permutation
problem not unlike (17) There are many papers (too numerous to mention) in
which complicated binomial sums are found by lattice point enumerations,, The
convolutions in 2, 3,4, 5J may mostly be found by such counting methods,, We
also note the recent papers of Greenwood |jf] and Stocks [ll] wherein the
Fibonacci numbers occur 0
The convolution addition theorem [2, 3,4, 5] of H Ae Rothe (1793)

(20)

^ A k ( a , b ) A n _ k ( c s b ) - A n (a + c,b) ,
k=o

valid for all real or complex a, b, c (being a polynomial identity in these), has
been derived several times by lattice point methods e We mention only a novel

1967] OF TERQUEM AND AN EXTENSION OF FIBONACCI NUMBERS

67

derivation by Lyness [13j 0 Relation (20) has been rediscovered dozens of times
since 1793, and its application in probability., graph theory, analysis, and the
enumeration of flexagons, etc , shows that the theorem is very usefuL In fact,
it is a natural source of binomial identities,, We should like to raise the question here as to whether any analogous relation involving the generalized Terquem
coefficients (4) existSo It seems appropriate to study the generating function
defined by

T(x;a,b) = J^

(21)

fa + (b - l ) n 1 \

n=o
for as general a and b as possible., If b is a natural number and a is an
integer >0, the series terminates with that term where n = a, as is evident
from the fact that a + (b - l)n < bn for n > a and the fact that |

1 = 0 for

k < n when n > 0, provided k > 0o We also note that for arbitrary complex
a and | x | < 1

T(x;a,l) =

2 () *" - {1+ x)"


n=o * '

so that in this case we do have an addition theorem:


T(x;a 9 l)T(x;c 5 l) = T(x ; a + c , l ) .
This, of course, corresponds to the case b = 0 in formula (20); the relation
implies the familiar Vandermonde convolution or addition theorem.
There does not seem to be any especially simple closed sum for the series
n
(22)

C n (a,c,b) -

which occurs in

^
k=o

/ f a + ( b - l ) k l \ / f c + (b - l')(n - k)] \

68

LATTICE POINT SOLUTION OF THE GENERALIZED PROBLEM


OF TERQUEM AND AN EXTENSION OF FIBONACCI NUMBERS
T(x;a,b)T(x,c,b) = V

xnCn(asc?b)

Feb

n=o
for a r b i t r a r y b 6

REFERENCES
1.

A Cayley, "On the Sums of C e r t a i n S e r i e s A r i s i n g f r o m the


x = u + f x , " Quart, J , M a t h . , 2(1857), 167-171.

Equation

2.

Ho Wo Gould, "Some G e n e r a l i z a t i o n s of Vandermond 1 s Convolution," A m e r .


Math, Monthly, 63(1956), 8 4 - 9 1 .

3e

H0 Wo Gould, " F i n a l A n a l y s i s of Vandermonde T s


Math. Monthly, 64(1957), 409-415.

4C

Ho W Gould, " G e n e r a l i z a t i o n of a T h e o r e m of J e n s e n Concerning Convolut i o n s , " D u ^ e M a t h o J o , 27(1960), 71-76 c

5.

Ho W0 Gould, "A New Convolution F o r m u l a and Some New Orthogonal


Relations for Inversion of S e r i e s , " Duke Matho J 0 , 29(1962), 393-404 o

6.

R0 Ec Greenwood, " L a t t i c e P a t h s and F i b o n a c c i N u m b e r s , "


Q u a r t e r l y , VoL 2, 1964, pp. 13-14.

7.

Po A. MacMahon, Combinatory A n a l y s i s , C a m b r i d g e , 1915-16, 2 V o l s . ,


Chelsea R e p r i n t , Ne Y. 1960 o

8.

E. Netto, Lehrbuch d e r Combinatorik,


R e p r i n t , N. Y. , 1958 0

Convolution,"

Leipzig, 1927, 2nd E d . ,

Amer.

Fibonacci

Chelsea

9 J . A. R a a b , "A G e n e r a l i z a t i o n of the Connection Between the F i b o n a c c i


Sequence and P a s c a l ' s T r i a n g l e , F i b o n a c c i Q u a r t e r l y , Vol 1, 1963, No
3, pp 0 2 1 - 3 1 .
10o

J . R i o r d a n , An Introduction to Combinatorial A n a l y s i s , N0 Y, 1958.

11.

D0 Ro Stocks, J r c , "Concerning L a t t i c e P a t h s and F i b o n a c c i N u m b e r s , "


F i b o n a c c i Q u a r t e r l y , VoL 3, 1965, pp c 143-145 0

12o

Oo T e r q u e m , "Sur un symbole c o m b i n a t o i r e d f E u l e r et son Utilite dans


P A n a l y s e , " J 0 Math 0 P u r e s Appl. , 4(1839), 177-184 0

13.

Ro Co L y n e s s , "Al Capone and t h e Death R a y , " Matho G a z e t t e ,


283-287o
* *** *

25(1941),

ADVANCED PROBLEMS AND SOLUTIONS


Edited by V. E. HOGGATT, JR., San Jose State College, San Jose, Calif.

Send all communications concerning Advanced Problems and Solutions to


Raymond Whitney, Mathematics Department, Lock Haven State College, Lock
Haven, Pennsylvania. This department especially welcomes problems believed
to be new or extending old results* Proposers should submit solutions or other
information that will assist the editor. To facilitate their consideration, solutions should be submitted on separate signed sheets within three months after
publication of the problems.
NOTICE;

H-103

PLEASE SEND ALL SOLUTIONS AND NEW PROPOSALS TO


PROFESSOR RAYMOND E9 WHITNEY, MATHEMATICS DEPARTMENT, LOCK HAVEN STATE COLLEGE, LOCK HAVEN, PA0

Proposed by David Z e i t l i n , Minneapolis, Minnesota


Show that

n
8

== FF

F 1

'3k+I
3k+I 3k+2
3k+2 66k+3
-'~

~3n+3

k=o
H-104

Proposed by VernerEo Hoggatt, J r s , San Jose State College, San Jose, Calif 0
Show
L

mX
m+

1 - 5Fm X ^ <-l) W
where L m andm F
H-105

are the m

k=o

5 (

^ mk

+ XL

^Dm)X2k '

Lucas and Fibonacci numbers,> respectively.


r
J

Proposed by Edgar Karst, Norman, Oklahoma, and S O c Rorem, Davenport,


Iowa,
Show for all positive integral n and primes p > 2 that

(n + 1) P - n P = 6N + 1
where N is a positive integer.

Generalize.
69

70

ADVANCED PROBLEMS AND SOLUTIONS

Feb.

H-106 Proposed by L. Carlitz, Duke University, Durham, N Carolina,

Show that

a)
k=o
k=0

Xx

'

k=o
k=0

H-107

'

'

T-^

/ \ 2

k=o

b)

'

k=o

Proposed by Vladimir Ivanoff, San Carlos, California.

Show that
F

p+2n

H-108

p+n

F ^0
q+2n

F ^
q+n

r+2n
for all integers p, q, r,

r+n

p
q

| - 0

and n.

Proposed by H. E, Huntley, Hutton, Somerset, U . K .

Find the sides of a tetrahedron, the faces of which are all scalene t r i angles similar to each other, and having sides of integral lengths.
H-109 Proposed by George Ledin, Jr, San Francisco, California*

Solve
X2 + Y2 + l = 3XY
for all integral solutions and consequently derive the identity:
F2
+ F2
+ l = 3F
F
*6k+7
*6k+5
-1
^6k+7^6k+5

"

H-110 Proposed by George Ledin, J r . , Son Francisco, California

Evaluate the double sum


n
m=i

oo
l^ j-m
k=i |_ k J

1967 ]

ADVANCED PROBLEMS AND SOLUTIONS

71

w h e r e [ x ] i s the g r e a t e s t integer in x9
H-lll

Proposed by John L. Brown, J r . , Pennsylvania State University, State


College, Pennsylvania.
Show that

[C

L n = "g" j l + 4 cos 2 ^ L z J :

m l

for n > 1 .

H-112 Proposed by L. Carlifz, Duke University, Durham, N . Carolina.


Show that, f o r n > 1,
a)

L*
- L 5 - L5
=
n+i
n
n-i

F^
- F5 - F5
= 5F
F F ,(2F 2 + ( - l ) n )
n+i
n
n-i
n+l n n - i
n

c)

L7
- L7 - L7
= 7L
T L
(2L2 - 5 ( - l ) n ) 2
n+l
n
n-i
n+l n n - i
n
F7
- F 7 - F 7 , = 7F
F F
(2F 2 + ( - l ) n ) 2
n+l
n
n-l
n+l n n - l
n

d)

5L

n+i n n - i

(2L2

- 5(-l)n)

SOLUTIONS
NO SOLUTIONS RECEIVED
H-59 Proposed by D. W . Robinson, Brigham Young University, Provo, Utah*
Show that, if m > 2,

then t h e p e r i o d of the Fibonacci sequence 0, 1,

1, 2, 3, - , F , * r e d u c e d modulo m is t w i c e t h e l e a s t p o s i t i v e i n t e g e r

such that F ,4 = ( ~ l ) n F . (mod m ) .


n+l
n-l
H-60 Proposed by Verner E. Hoggatt, Son Jose State College, San Jose, Calif.
It i s well known that if p,
k

mod 1 0 ,

then pt = 60,

is the l e a s t i n t e g e r such that F

p 2 = 300 and p k = 1,5 x 1 0

Fibonacci, then for fixed k,

, = F

for k > 3. If Q(n,k)

i s the k

digit of the n

Q(n, k) i s p e r i o d i c ,

that i s q,

is the l e a s t integer such that Q(n + q , , k ) = Q(n, k) mod 10.

an explicit e x p r e s s i o n for q, .
H-62 Proposed by H. W Gould, W Virginia University, Morgantown, West
Virginia (corrected).
Find all polynomials f(x) and g(x),

of the f o r m

Find

72

ADVANCED PROBLEMS AND SOLUTIONS


f(x + 1) = 2__j a - x i

an

[Feb.

integer

3=0
s
g(x) = Y b . x ,

b . an i n t e g e r

3=o

such that
2Jx 2 f 3 (x + 1) - (x + l) 2 g 3 (x)} + 3{x 2 f 2 (x + 1) - ( x + l) 2 g 2 (x)}
+ (2x + l){xf(x + 1) - (x + l)g(x)} = 0 .
LIMIT OF LIMITS
H-61 Proposed by P, F. Byrd, San Jose State College, San Jose, Calif.(corrected)
Let
f

=0

for

0 < n < k - 2,

f,

, , = 1

and

k
f

, = / f .,
n?k
-** n - j , k
3=i

for

n > k

Show that

n+i,k
Hence
f
lim

Hm

'
f

k( n-8^oo

_!.< 1

'

n+i,k

Solution by Douglas Lind, University of Virginia, Charlottesville, Virginia.


The sequence if

obeys a r e c u r r e n c e whose a u x i l i a r y polynomial

is
l

f (x) = x
Let r

1 ,
ljK.

r , , e , r, ,
2j]

k-i
-x

k-2
- x

.
- - x - 1

be the k r o o t s of f(x) = 0 The k initial conditions

JijK.

given d e t e r m i n e constants b , , b , , * " , b, ,

such that

1967]

ADVANCED PROBLEMS AND SOLUTIONS

73

k
nsk

*->* j 9 k ],k

Now Miles [''Generalized Fibonacci N u m b e r s and A s s o c i a t e d M a t r i c e s , 1 ' A m e r .


Math. Monthly, Vol. 67, pp. 7 4 5 - 5 7 ] h a s shown that all but one of the r o o t s
r. ,

lie within t h e unit c i r c l e ,

f(l) = 1 - k < 0,

f(2) = 1,

so that

jr. , 1 < 1 (1 < j < k).

and s i n c e f i s continuous,

Note

that

the r e m a i n i n g

root

r, T m u s t be a r e a l n u m b e r between 1 and 2 . Then b, , ^ 0, b e c a u s e


k,k
k,k
n
l i m r . , = 0 (1 < j < k) while l i m f . = oo. We also have
n
r. ,
lim - i ^ = 0

(1 < j < k) ,

k,k
so t h a t
lim
^oo

, /f

'

k /

,J ,
n+1 k

'

I V b. , r11,
/
Y! b. , r. ,
A
= M/ 1/r,,
k,k "
\PL
^ k J' k // \ j-i J'k ^k k , I

lim
n^oo

We have a l r e a d y shown r, , < 2.

Now

(2k) k - (2k) k " 1 (k + 1) - (2k) k " 2 (k + 1)2 - - - - (2k)(k + l ) k " A - (k + l ) k


< (2k) k - ( 2 k ) k - 1 k - (2k) k " 2 k - . . . - (2k)k k _ 1 - k k - k k
= 2 k k k - k k ( 2 k _ 1 + 2 k " 2 + + 2 + 1 + 1) = 0
and division by (k-t 1)

shows

f
Since

(m)<

i < j t < i
we have 2 > r, , > 2k/(k + 1),

and i n v e r s i o n gives t h e f i r s t r e s u l t of the

JtC,.K

problem.

The second r e s u l t follows by taking l i m i t s a s

k.

ODD ROW SUMS OF FIBONOMIAL COEFFICIENTS


H-63 Proposed by Stephen J e r b i c , Son Jose State College, San Jose, California.
Let
F F
F
m
m 1
-n/ A\ -i
J -n/
\
~
m-n+i A .
.
F(m,0) = 1 and F(m 5 n) = ^
=; 0 < n < m ,
x
n n-l

74

ADVANCED PROBLEMS AND SOLUTIONS

[Feb.

b e the Fibonomial coefficients, w h e r e F

is the n
Fibonacci n u m b e r . Show
n
2m-i
m-l
]T F(2m - l , n ) = 0 L 2 i ,
m > 1 .
n=o
i=o
Solution by Douglas Lind, University of Virginia, Charlottesville, Virginia.
n
Put
Sn = F<n>r)
r=o
and
(1)

= ^(-l)r(r+l)/2F(n,r)xn-r
r=o

f n (x)

B r e n n a n ( n Fibonacci P o w e r s and P a s c a l ' s T r i a n g l e in a M a t r i x , ? t Fibonacci


Q u a r t e r l y , Vol. 2, No. 2S pp. 93-103) h a s shown
f x(x) - (-l) n ~ 2 (x 2 - L
x + (-if"1)!
J-x)
n '
n-i
n-2 x
i s the n
Lucas n u m b e r . Setting x = \ / ^ l 7 . n = 2m + 1, we find

where L

7i) = iL f
(-i)
2m+i
2m 2m-i

Using (1) this b e c o m e s


m
m
] F ( 2 m + 1, 2r) + i ]T F ^ 2 m
r=o
r=o
=

2m

*>

2r +

*)

m-i
m-l
E F < 2 m " 1. 2 r + 1) + i L 2 m J2 F ( 2 m - 1, 2r) ,
r=o
r=o

and so equating r e a l and i m a g i n a r y p a r t s , taking absolute v a l u e s , and adding


we get S
&

,
2m+i

= L

, which, with Si = 2 = L 0 ,
1

2 m 2H1-1

p r o v e s the proposition.
J-

ONE OF MANY FORMS


H-64 Proposed by Douglas Lind, University of Virginia, Charlottesville, Virginia.
Show

where F

F
is the n

n+i

ini(l-2ico

JiL)

Fibonacci n u m b e r .

Solution by David Z e i t l i n , Minneapolis, MinnesotaF o r a generalization, let W 0s W l s C ^ 0,

and d fi 0 be a r b i t r a r y r e a l

n u m b e r s , and define W , = dW _,. - cW , n = 0 , 1 , , with d2 - 4c f 0


n+2
n+i
n
We define V = W , n =" 0 , 1 , , when W0 = 1 and Wt = d; and set Z

1967]

ADVANCED PROBLEMS AND SOLUTIONS

75

= W , n = 0 , 1 , , when W0 = 0 and Wj. = 1* In terms of Chebyshevpolynomials of the first kind, T (x), and of the second kind U (x), it is readily
verified that
(1)

Z + 1 , cn/2U (-$-)
n+1

\2V5 /

e 2cn/2T

(-*-)

\2VH /

Since

u n - 2* n (x - cs J t t , ) , T W - 2 - n (x - cos <5ti)n),
we obtain from (1)

<2>

V .

(3)

-"*&?-'-*)

V.
n11 -

"

If d = 1 and c = - 1 , then Z = F
n
n

"

"

and V = L . Since - 1 = i 2 , we obn


n

tain from (2) and (3), r e s p e c t i v e l y , 1

. | ( -2i cos ^Hrr )

<4)
(5)

n+1

,.a(1.Me0BfiLiJ)n)
11

3
Also solved by F, D. Parker, John L Brown, J r . , and the proposer.

FIBONACCI RELATED NUMBER


H-65 Proposed by J . Wlodarski, Porz-Westhoven, Federal Republic of Germany*
The units digit of a positive integer s

M9 is 9e Take the 9 and put it on

the left of the remaining digits of M forming a new integer s Ns such that
N = 9Me Find the smallest M for which this is possible*
Solution by Robert H. Anglin, Danville, Va, and Murray Berg, Oakland, Calif*
n
M = 9 + X^x.10 1
i=i

76

ADVANCED PROBLEMS AND SOLUTIONS

[Feb.

n
N = x.10 1 + 9 - 1 0 n = 9M
1=1

ION = 90M = V x.10 1 + 9 0 - 1 0 n = M - 9 + 9 0 1 0 n


89M = 9 ( 1 0 n + 1 - 1)
M

= 9 ( l Q n + 1 - 1)
89

89999-991
89

By p e r f o r m i n g the actual division the f i r s t z e r o o c c u r s when the quotient i s


M = 1011 23595 50561 79775 28089 88764 04494 38202 24719
W l o d a r s k i notes
M = 10 43 +

10" V 2 k
m=l 10

w h e r e [ x ] i s the g r e a t e s t i n t e g e r function in x.
Also solved by Marjorie Bicknell, James Desmond, A . B. Western, J r . , C . B . A ,
Peck, and the proposer,

A STIRLING NUMBER SOLUTION


H-66

Proposed by Douglas Llnd, University of Virginia, Charlottesville, V a . , and


Raymond E. Whitney, Lock Haven State College, Lock Haven, Pennsylvania.
k
Let
V a.y ,. = 0

b e a l i n e a r homogeneous r e c u r r e n c e r e l a t i o n with constant coefficients

a,.

Let the r o o t s of the auxiliary polynomial


k

Lax3 = 0
j=o
be

rl9 r2, o o , r m

and each root r j be of multiplicity m^ (i = 1,2, , m ) .

J e s k e (Linear R e c u r r e n c e Relations P a r t I, Fibonacci Q u a r t e r l y ,


No. 2, pp. 69-74) showed that
oo

,n

* = y

. m.-i
ri. t
l

b..tj .

Vol,

1967]

ADVANCED PROBLEMS AND SOLUTIONS

77

He a l s o stated that from t h i s we m a y obtain


m.-i
m r .__ Vi
= V
~
l A-*
1=1
3=0

Jy

<*)

(i) Show that (*) i s in g e n e r a l i n c o r r e c t s

b..ir
ij

(ii) s t a t e u n d e r what conditions it yields

t h e c o r r e c t r e s u l t , and (iii) give the c o r r e c t formulation,,


Solution by the proposers
Let s. = m . - 1,
i

and put
^

oo

YW

,n

r.t

= E w = E * " EYbf .

r
n=o n m
i=i
i=o
Now define n , v = n(n - ! ) (n - s + 1), n , ^ = 1,
"k
Y k (t) = e

E b,kj.t

so that

1J

and for k = 1, 2, , m let

j=o
oo
.(t) =

E E b,kj.

v!

v=o j=o

oo S k
= E E b ki
r>
Wv

'(j)

V=0 ]=0

F o r p = 0 , 1 , s s,

v+j
*
F^TJ!

T
+ JJ ) . .

put
oo

Ykk-npft) .= v=o
EVk(v

+ P)

f v+p

(P) ^rw

Differentiating t h i s n t i m e s and setting t = 0,


/ v
y W (0)
k,p

,v+p-n
,
n-p
n,,
-P\
t
_^
= b, r, ^ n , x = r, (b, n , x r, ^ ) .
^ ( p ) (v + p - n)! t=o
kp k
(p)
k v kp (p) k
T h u s applying the i n v e r s e t r a n s f o r m (393)9 we find
=

oo
y b
v(v
^
kp k x

yn = Y(n)(0) = g

s,

E YW (0) =

k=ip=o

^sP

g rn ^
i=i

j=0

-J
X

J {V

78

ADVANCED PROBLEMS AND SOLUTIONS

[Feb.

which is the correct form.


(ii) If m. = 1 (i = 1,2, , ! ) , then since n,^

= n, Jeske f s form gives the

correct result. Also, since n^x = n 1 , his result will be correct if all roots of
multiplicity two are one, and there are no roots of greater multiplicity.

For

higher multiplicities his form almost never gives the correct result.
(i) We need only take a recurrence whose auxiliary equation does not satisfy
the conditions of (ii) to form a counterexample to (*).
Also solved by P. F. Byrd and D. Z e i t l i n .

Editorial Comment; The b.. in the first displayed equation above are arbitrary
constants. The b.. in the second displayed equation are also arbitrary constants. In this sense Jeske is correct. However, most readers would probably
incorrectly infer that after you have determined the specific constants for a
given problem one can then use these in the second displayed equation which, of
course, is not true in all cases.

V. E EL

AN INTERESTING ANGLE
H-67 Proposed by J . W . Gootherts, Sunnyvale, California.

Let B = ( Bus0 , Bl9i , - - - , B ) and V = (F , F , . . . . , F , ) be two vectors


n
m m+i
m+n
in Euclidian n + 1 space. The B. f s are binomial coefficients of degree n and
the F

.Ts are consecutive Fibonacci numbers starting at any integer m.


Find the limit of the angle between these vectors as n approaches infinity.

Solution by F. D. Parker, Sony at Buffalo, N . Y .

We start with the formula


cos2e =

S^JDL ,
|B|1V|*

where B - V is the scalar product of B and V, and | B | , | v | represent the


magnitudes of B and V,

respectively.

The following results are easy to verify by mathematical induction:


(1)
x ;

(2)

B. V = F ,
m+2n
|B| =

v^)

1967]

ADVANCED PROBLEMS AND SOLUTIONS

(3)
v
'

79

| V | = \/F
, F , ^ - F
F
v
I I
m+n m+n+i
m-i m

Thus

.
[B-V|2

|B|

(F

|8V |p2

2
m+2n)

2n

lI n) I v(Fm+n
- F m - i F~~)
A F m+n+i
M ^
m7

But

lim
n#-oo

)2

(F

m+n m+n+i

lim / 2 n \
n-^oo\n J

m-i

, ,
and hence

= GO
'

l i m

and

Y~

l i m c o s # = 0,
'
n-^-Qo

and

0 = ,r/2

n -#-oo
Also solved by the proposer.
MANY ROADS TO MORGANTOWN
H-68 Proposed by H. W . Gould, W , Virginia University, Morgonfown, W . Va
P r o v e that
n

n^
>

k=i k
with equality only for n = 1, 2

_ !

. n > 1

n+2

Solution by the proposer,


T h e well-known identity

E ^ E B jJ = nEA.B. + | t t (\ - AJ )(B
- B.)
J

i=i

j=i

i=i

i=i j=i

yields the special case


n

i=i

j=i

(A. - A . ) 2
i

i=ij=i

3
i 3

whence it i s evident that for p o s i t i v e A f s we have the inequality


n

1-1

j=i

ADVANCED PROBLEMS AND SOLUTIONS

80

F e b . 1967

with equality only when A. = A. for all 1 < i < n, 1 < j < n. T h e application
to the Fibonacci n u m b e r s F (with F ( J = F +F a and F i = 1, Fo = 1 )
n
n+i
n
n-l
i
* >
i s evident from the formula
F

i = Fn+2 "

1=1

so that we find
n

y l >

_B!

.^J F . " F

i=i

n+2

- 1

with equality only for n = 1, 2.


Zeiti m and Desmond used the Arithmetic-Harmonic mean inequality. Brown used
the Schwarz inequality*
Further results are:

n2

fi B; * H
k=i
A

- H2 n - x

n+2

i - > =-5?

h L

(Zeitlin)

n n+i

n >1

(Hoggatt)

Also solved by D. Z e i t l i n , John L Brown, Jr.- A/LN.S. Swamy, D. Lind, C . B . A .


Peck, and John Wessner.

SOME BELATED SOLVERS' CREDITS


H-37 Dermott A . Breault
H-48 John L. Brown, J r . , and Charles R. Wall
H-52 C . B . A . Peck, F. D. Parker, and D. Lind
H-57 John L. Brown, J r . , Charles R. W a i l , MarjorieBicknell, F.D. Parker, and
M . N . S . Swamy
H-58 David Klarner
H-74 John I , Brown, J r .

*****
Continued f r o m page 44.
REFERENCE
1.

S. L . B a s i n and V. E. Hoggatt, J r . , T,A P r i m e r on the Fibonacci Sequence


- P a r t II, " Fibonacci Q u a r t e r l y , Vol. 1 (1963), No. 2 , 61-68.

*****

RELATIONS INVOLVING LATTICE PATHS AND CERTAIN SEQUENCES OF INTEGERS


DAVID R. STOCKS, J R . , Arlington State College, Arlington, Texas

Relations involving c e r t a i n s p e c i a l p l a n a r l a t t i c e paths and

certain

s e q u e n c e s of i n t e g e r s have been studied p r e v i o u s l y [ 1 ] , [ 2 ] . We will s t a t e c e r tain b a s i c definitions which p e r t a i n to t h e s e s t u d i e s , develop additional r e s u l t s


involving other p l a n a r lattice p a t h s , and finally, indicate g e n e r a l i z a t i o n s of
t h e s e r e s u l t s for l a t t i c e p a t h s in k dimensional s p a c e .

F o r convenience of

r e f e r e n c e s o m e of the definitions a r e collected t o g e t h e r and p r e s e n t e d in P a r t 1.


The r e m a i n i n g m a t e r i a l will be found in P a r t 2.
Part 1
In Euclidean k - d i m e n s i o n a l space the s e t X of points such that p belongs
to X if and only if each coordinate of p i s an i n t e g e r i s called the unit l a t t i c e
of that s p a c e .
The s t a t e m e n t that P i s a l a t t i c e path in a c e r t a i n s p a c e m e a n s that P
i s a sequence such that
1) each t e r m of P i s a m e m b e r of the unit l a t t i c e of that s p a c e , and
2) if X i s a t e r m of P and Y i s the next t e r m of P and x^ and yi
a r e the i

c o o r d i n a t e s of X and Y r e s p e c t i v e l y , then ]x^ - y j

1 o r 0 and for s o m e j , |XJ - y j | = 1.


If each of X and Y i s a point of the unit l a t t i c e in Euclidean k - d i m e n s i o n a l
s p a c e , then the s t a t e m e n t that the l a t t i c e path P i s a path from X to Y m e a n s
that P i s finite,

X i s the f i r s t t e r m of P, and Y i s the l a s t t e r m of P 0

P i s a l a t t i c e path,
by the step \XtY]

X i s a t e r m of P, and Y i s the next t e r m of P ,

If

then

of P i s meant the line i n t e r v a l whose end points a r e X

and Y.
A l a t t i c e path P in Euclidean 2 o r 3 - s p a c e i s said to be s y m m e t r i c with
r e s p e c t to the line k if and only if it i s t r u e that if X i s a point of s o m e s t e p
of P ,

then e i t h e r X i s a point of

k or t h e r e e x i s t s a point Y of s o m e step

of P such that k i s the p e r p e n d i c u l a r b i s e c t o r of the line i n t e r v a l


Suppose that S = [ (xi9yt),

(x 2 ,y 2 )] i s a step of

s o m e l a t t i c e path P in

Euclidean 2 - s p a c e 9 S i s said to be x - i n c r e a s i n g if x 2 -xt


81

[ X , Y ].

= 1 and x - d e c r e a s i n g

82

RELATIONS INVOLVING LATTICE PATHS

if x 2 - xt = - 1 . The t e r m s
defined,

y-increasing

and

[Feb.

y-decreasing

are similarly

A s t e p i s s a i d to be x y - i n e r e a s i n g if it i s both x - i n c r e a s i n g

y-increasing.

To say that

S is

x - i n c r e a s i n g only m e a n s that S i s

i n c r e a s i n g but n e i t h e r y - i n c r e a s i n g n o r y - d e c r e a s i n g
monotonically i n c r e a s i n g

P i s said to be

if and only if it i s t r u e that if

then 2 i s not x - d e c r e a s i n g .

2 i s a step of

A s t e p 2 i s said to be horizontal if it i s n e i t h e r

nor y-decreasing.
m e a n s that

xxP,

The t e r m y-monotonically i n c r e a s i n g i s s i m i -

l a r l y defined. A s t e p 2 i s said to be v e r t i c a l if it i s n e i t h e r x - i n c r e a s i n g
x-decreasing.

and

nor

y-increasing

The s t a t e m e n t that the path P i s duotonically i n c r e a s i n g

i s both

x-monotonically i n c r e a s i n g

and

y-monotonically

increasing.
Part 2
In Euclidean 2 - s p a c e a path from

(0,0) to (n,n) i s said to have p r o p e r t y

G if and only if:


1) it i s duotonically i n c r e a s i n g ,
2) it i s s y m m e t r i c with r e s p e c t to the line x + y = n,
3) no s t e p of it which contains a point below the line

and
x + y = n

is

vertical.
A path having p r o p e r t y G will be called a

G-path.

T h e o r e m 1 (Greenwood)
Let g(0) = 1 and g(l) = 1.

F o r each positive i n t e g e r n > 2, l e t g(n)

denote the n u m b e r of G-paths from (0,0) to (n - 1, n - 1).

The sequence

{g(0)> g U h 8 J g( n ) J } i s the Fibonacci sequence.


Proof. By definition g(0) = g(l) = 1. Suppose n = 2. The only G-paths
f r o m (0,0) to (1,1) a r e

{(0,0), (1,0), (1,1)} and {(0,0), (1,1)}, thus g(2) = 2.

F o r n = 3, the G-paths from (0,0) to (2,2) a r e


{(0,0), (1,0), (2,1), (2,2)} and
Suppose

n > 4.

{(0,0), (1,1), ( 2 , 2 ) } , so that g(3) = 3.

Each

initial s t e p e i t h e r [ (0,0), (1,0)]


step

{(0,0), (1,0), (2,0), (2,1),(2,2)} 9

G-path
or

from

(0,0) to (n - 1, n - 1) h a s a s i t s

[(0,0), (1,1)]. If a G-path h a s a s i t s initial

[(0,0), (1,0)], then 5 because of symmetry, i t s t e r m i n a l s t e p i s [(n - 1 , n - 2),

(n - 1, n - 1 ) ] ; and t h u s it contains a s a subsequence a G-path from (1,0) to


(n - 1, n - 2).

But the n u m b e r of G-paths from

n u m b e r of G-paths from

(0,0) to (n - 2, n - 2),

(1,0) to (n - l , n - 2) i s the
i. e. , g(n - 1).

Likewise, if a G-path h a s a s i t s initial s t e p


terminal step is

[(0,0), (1,1) ],

then i t s

[(n - 2, n - 2), (n - 1 , n - l ) ] > a n d i t contains a s a subsequence

1967]

AND CERTAIN SEQUENCES OF INTEGERS

a G-path from (1,1) to (n - 2, n - 2).


n u m b e r of

G-paths

from

(0,0)

to

83

The n u m b e r of such G-paths i s the

(n - 3, n - 3), which is

g(n - 2)0

Thus

g(n) = g(n - 1) + g(n - 2).


The s t a t e m e n t that a path in Euclidean 2 - s p a c e has p r o p e r t y H m e a n s
that it h a s p r o p e r t y G and i s such that one of i t s t e r m s belongs to the line
x + y = n.

A. path having p r o p e r t y H will be called an

H-path.

Obviously, if n i s a positive i n t e g e r , then the s e t of all H-paths


(0,0) to (n,n) i s a p r o p e r s u b s e t of the s e t of all G-paths from

from

(0,0)

to

(n,n); y e t , u s i n g an a r g u m e n t s i m i l a r to the above, we may e s t a b l i s h

the

following.
T h e o r e m 2
Let h(0) = 1 and,for each positive i n t e g e r n, let h(n) denote the n u m b e r of H - p a t h s from (0,0) to (n,n).

The sequence {h(0),h(l), ,h(n), . }

i s the Fibonacci sequence,,


An obvious but i n t e r e s t i n g c o r o l l a r y i s that the n u m b e r of H-paths from
(0,0) to (n,n) i s the n u m b e r of G-paths from (0,0) to (n - 1, n - 1).
Greenwood h a s d i s c u s s e d G-paths [1J .

A method of e n u m e r a t i o n

dif-

f e r e n t from that used by Greenwood l e a d s to the following [ 2 ] ,


T h e o r e m 3.
Let
z(l,i)
z(2,i) =

i - 1

, where

[ ] denotes the g r e a t e s t i n t e g e r function,

z(3,i) = z(3,i - 1) + z(2,i - 1)


z(4,i) = z ( 4 5 i - 2 ) + z ( 3 , i - 2 )

z(2n,i) = z ( 2 n , i - 2) + z(2n - l , i - 2) ,
z(2n+l,i)

= z ( 2 n + l , i - 1) + z ( 2 n , i - 1)

84

RELATIONS INVOLVING LATTICE PATHS

with the restriction that z(k,i) = 0 if k > i.

[Feb.

For each positive integer i,

let
i

f(i) = X/ ( k j i )
fc=i

The sequence {f(i) |i = 1, 2, } is the Fibonacci sequence.


The proof is direct and is omitted.

A geometric interpretation of the

numbers z(k,i) and f(i) is given in [2],


It is interesting to note the sequence obtained by considering paths in
3-space that are analogous to H-paths in 2-space.

In Euclidean 3-space, a

path from (0,0,0) to (n,n,n) is said to have property F if and only if it is


such that:
1) it is symmetric with respect to the line z = (n/2) in the plane x +
y = n ,
2) if the step [ P l 9 P 2 ] of it is z-increasing only, then P 1 belongs to the
plane x + y = n ,
3) if S is a step of it which is not z-increasing only, then either S is
x-increasing only, y-inereasing only, or xyz-increasing, and
4) some term of it belongs to the plane x + y = n.
We will call a path an F-path if it has a property F
We define f(0) = 1; andsfor each positive integer n, let f(n) denote the
number of F-paths from (0,0,0) to (n,n,n)c We note that f(l) = 2 and f(2)
= 5. If n > 2, then each F-path has as its second term either (1,0,0),
(0,1,0), or (1,1,1). If an F-path from (0,0,0) to (n,n,n) has as its second
term (1,0,0) or (0,1,0), then it has as its next to last term (n,n - 1, n)
(n - 1, n, n) respectively.

or

The number of F-paths from (0,0,0) to (n,n,n)

which have as their second term either (0,1,0) or (1,0,0) is the number of
F-paths from (0,0,0) to (n - 1, n - 1, n - 1). Hence,the number of F-paths
from (0,0,0) to (n,n,n) whose second term is either

(1,0,0) or (0,1,0)

is

2f(n - 1). Similarly, the number of F-paths from (0,0,0) to (n,n,n) whose
second term is (1,1,1) is f(n - 2). Hence, if n > 2, then f(n) = 2f(n - 1) +
f(n - 2).

1967]

AND CERTAIN SEQUENCES OF INTEGERS

85

It i s noted that the e x p r e s s i o n f(n) = 2f(n - 1) + f(n - 2) i s the s p e c i a l


c a s e of the Fibonacci polynomial

f (x) = xf _ (x) + f

(x)

for

f0(x) = 0,

fj(x) = 1, and x = 2.
Using the methods of finite difference equations we may obtain an e x p r e s sion for calculating f(n) directly,,

C o n s i d e r again the r e c u r s i o n r e l a t i o n

= 2f(n - 1) + f(n - 2) in the f o r m of the second o r d e r homogeneous

f(n)

difference

equation
f(n + 2) - 2f(n + 1) - f(n) = 0 '.
The c o r r e s p o n d i n g c h a r a c t e r i s t i c equation
r 2 - 2r - 1 = 0
has roots
rt

= 1 + \f2

and

r 2 = 1 - <{2 .

The g e n e r a l solution of the above difference equation is


f(n) = Ci(l + N/"2)n + C 2 (l - -\f2) n .
Using the initial conditions of f(0) = 1 and f(l) = 2, the c o n s t a n t s

Ct

and

C 2 a r e found to be
(A/2 + l)/2\l2

and

(N/2 -

l)/2\[2

r e s p e c t i v e l y , so that we have finally

f(n)

(1 + ^ ) n + 1 _ ; (1 - ^ )
2 \[2

n + 1

An a n a l y s i s s i m i l a r to that used to obtain the r e c u r s i o n r e l a t i o n

for

F - p a t h s in 3 - s p a c e suffices to show that in k - d i m e n s i o n a l s p a c e the n u m b e r


of p a t h s from

(0,0,0,- ,0) to (n,n,n, ,n)

that a r e analogous to F p a t h s

in 3 - s p a c e s a t i s f i e s the r e c u r s i o n r e l a t i o n f(n) = (k - l)f(n - 1) + f(n - k + 1).

RELATIONS INVOLVING LATTICE PATHS


AND CERTAIN SEQUENCES OF INTEGERS

86

Feb. 1967

REFERENCES
1. R. E. Greenwood, "Lattice Paths and Fibonacci Numbers, n The Fibonacci
Quarterly, Vol0 2, No. 1, pp. 13-14.
2. D. R. Stocks, J r . , "Concerning Lattice Paths and Fibonacci Numbers,"
The Fibonacci Quarterly, Vol. 3, No. 2, pp. 143-145.
3.

C. Jordan,

Calculus of Finite Differences,

2nd ed. New York: Chelsea

Publishing Company, 1947.


The references shown below are for "Iterated Fibonacci and Lucas Subscripts,"
which appears on page 89.

REFERENCES
1. EL H Ferns, Solution to Problem B-429 Fibonacci Quarterly, 2 (1964),
No, 4, p0 329.
20 I. D RugglesandV. E Hoggatt, Jr a , "A Primer on the Fibonacci Sequence, "
Fibonacci Quarterly, 1(1963), No 4, pp. 64-71
30 Raymond Whitney, Problem H-55, Fibonacci Quarterly,
Po 450
^

3(1965), No. 1,

The Fibonacci Bibliographical Research Center desires that any reader


finding a Fibonacci reference send a card giving the reference and a
description of the contents. Please forward all such information to:

brief

Fibonacci Bibliographical Research Center,


Mathematics Department,
San Jose State College,
San Jose, California

The Fibonacci Association invites Educational Institutions to apply for academic


Membership in the Association. The minimum subscription fee is $25 annually.
(Academic Members will receive two copies of each issue and will have their
names listed in the Journal.)

CHAINS OF EQUIVALENT FIBONACCI-WISE TRIANGLES,


DEWEY C. D U N C A N , Los Angeles, California

Consider the infinite set of o r d e r e d and equally d i s p e r s e d Fibonacci n u m bers,

n+ii1

i = 0, 1, 2, , n , h, a r b i t r a r y positive i n t e g e r s . The t r i a n g l e

having v e r t i c e s at the points designated by the r e c t a n g u l a r c a r t e s i a n c o o r d i n a t e s

(F

n> W '

(F

n+2h s

nH-3h )s

(F

n+4h s

n+5h'

F
+ F F
- F F
- F F
2h sh
h 4h
sh 4h
*h2h

has the a r e a

which i s noted to b e independent of n and depends only upon the d i s p e r s i o n of


t h e Fibonacci n u m b e r s u s e d for c o o r d i n a t e s of the v e r t i c e s *
PROOF
Twice the a r e a of t h e specified t r i a n g l e i s equal to the absolute value of
the d e t e r m i n a n t
"n+h
" n+3h

'n+2h
" n+4h

n+5h

whose expanded f o r m , simplified by the identity


F
a+b

= F F
+ F
F
b a+i
b-i a

r e d u c e s to
AF*
+ BF
F + CF 2
n+i
n+l n
n
wherein
A = F ,F , + F,F. - F , F , - F,F.
,
2h 5h
h 4h
3h 4h
h 2h
B = F F
+ F F
+F
F
+ F ,
15
Jj
2h 5h-l
5h 2h-i
hr4h
*3h
F F
- F F
- F
- F F
sh 4h-i
r
4li 3h-i
h 2h-i
5h
C = F
F
+ F
F
+ F
u
2hi 5h-i
H-Ah-i
3h-i
F
F
- F
F
5h-i
*3h-r4h-i
h - i 2h-i
By u s e of the identity cited above 3 the fundamental r e l a t i o n s h i p
F

one m a y easily p r o v e that A = - B = - C .


F2
- F
F
n+i
n+i n

F2
n

F u r t h e r m o r e * since
1

t h e a r e a of t h e t r i a n g l e Is o b s e r v e d to b0e half the value of


87

+ F

n+i

88

CHAINS OF EQUIVALENT FIBONACCI-WISE TRIANGLES


F

2hF5h

hF4h -

hF2h -

3hF4h

F e b . 1967

E - D

COROLLARIES
lo

F o r any p o s i t i v e i n t e g r a l value of h t h e r e a r e 2h chains of F i b o n a c c i - w i s e

t r i a n g l e s ; L e . , t r i a n g l e s of equal a r e a extending along the two s e r i e s of v e r t e x


points whose r e c t a n g u l a r c a r t e s i a n c o o r d i n a t e s a r e equally d i s p e r s e d Fibonacci
numbers.

In each chain consecutive t r i a n g l e s have two v e r t i c e s in common.

2.

By exhibiting t h e fundamental r e l a t i o n s h i p of Fibonacci n u m b e r s as F

= F

, one may define the Fibonacci n u m b e r s for z e r o and negative i n d i c e s ,

to wit, F 0 = 0, F
= 1, F
= -1, F
= 2 , and quite g e n e r a l l y , F
n+i
1
2
3
n
(-1)
F . Accordingly, the 2h chains of F i b o n a c c i - w i s e t r i a n g l e s extend
indefinitely in both d i r e c t i o n s .
3.

Again, the Fibonacci r e l a t i o n s h i p


F
k

+ F
= F
*k+i
k+2

i s o b s e r v e d to b e valid for all r e a l v a l u e s of k for the added two compatible


definitions
F, = k
k

for

0 < k < 1,

and

F, = 1
k

for

1 < k < 2 .
~~

Hence one obtains a n o n - d e n u m e r a b l y infinite set of F i b o n a c c i - w i s e chains of


t r i a n g l e s for any p r e s c r i b e d positive i n t e g r a l value of h,

w h e r e i n individual

t r i a n g l e s of neighboring chains extend continuously along t h e s e t s of

real

F i b o n a c c i n u m b e r s employed a s r e c t a n g u l a r c a r t e s i a n c o o r d i n a t e s of v e r t i c e s .

ITERATED FIBONACCI AND LUCAS SUBSCRIPTS


D. A . U N D , University of V i r g i n i a , Charlorfesvilie, V a .

Raymond Whitney C 3 h a s p r o p o s e d t h e p r o b l e m of finding r e c u r r e n c e


r e l a t i o n s for the s e q u e n c e s U = FT? , V' = F T
W = L T ,J , and X =
J, . ,
^
n
*n
n
n'
n
-hi
n
L xp 9 w h e r e F and L a r e the i r " Fibonacci and Lucas n u m b e r s , r e s p e c t n
n
n
ively* In t h i s note we give the r e q u i r e d r e c u r r e n c e r e l a t i o n s for m o r e g e n e r a l
s e q u e n c e s of the f o r m Y

= Fg ,

= LJJ , where the H

a r e generalized

Fibonacci n u m b e r s introduced by H o r a d a m .
We will m a k e u s e of s e v e r a l i d e n t i t i e s .

It follows from t h e Binet f o r m s

for Fibonacci and Lucas n u m b e r s that


(1)

2F ^ = F + L
n+i
n
n

(2)

-F
= 4(L- - F ) ,
l
n-i
n
n

(3)

L 2 - 5F 2 = 4 ( - l ) n
n
n

(4)

2L ^ = 5F + L
o
n+l
n
n

F r o m t h e s e H H F e r n s 11 h a s shown
(5)

Fn+1 = |(vi5F^ + 4(-l)n + Fn)

(6)

Ln+1 = ^ 5 l

- 20(-l)n

Ln)

Equation (5) i m p l i e s
F Q _ I = {(sfo* + 4(-l)n - Fn)

(7)

We shall a l s o r e q u i r e
(8)
v
'
%

_,_ ^ = F F + F ^ . F , 4
m+n+l
m n
m+i n+l
m+n+l

which a r e found in [2; Section 5]


- 3fn2/3j9

m n

m+i n+i

Finally, it i s convenient to define s(n) = n 2

w h e r e [ ] denotes the g r e a t e s t i n t e g e r function,,

if 3Jn while s(n) = 0 if 3In,

it follows that
89

Since

s(n) =

90

ITERATED FIBONACCI AND LUCAS SUBSCRIPTS


,s(a)
(-1)"

(_l) n

F e b . 1967

(.D ^

F i r s t c o n s i d e r t h e sequence Y

= FTJ , w h e r e H obeys H , = H ,_
J
n
*%
n
n+2
n+l
+ H Then u s i n g (8), (7), and (5), we find
1

n+2

| F

H n -f 2

H n

( ^

Hn+i+Hn =

+ 4

Hn+rlFHn

Hn+1*Hn+i

( V ^ - F H n + i ) + i F H n + 1 (X/SF^ + 4(-l) H n

FHn)

n+l

5Y*
4 ( ~ l ) H n + 1 + Y ,_ V 5 Y 2 + 4 ( ~ l ) H n
n+l
n+l
n

H H = F 9 then Y = U and we h a v e
n
n
n
n
U N/SU2 +4(~l)s(n+i) + U ^>/5U2 + 4(-l)s(n)
n+i
n+l
n

Un+2

2 . n

(n > 0)

while if H = L , then Y = V and we find


n
n
n
n

n+ 2 = f[ V n V , 5 V n + 1 + 4 (- 1 ) S ( n + 1 )

Now c o n s i d e r t h e s e q u e n c e Z
(9),

\+/5K

+ 4<-l)i

= LJJ , where H

Mxiy

is as before.

(2), (3), and (6), we s e e


Z

n+2

Hn+2

Hn+1+Hn = F H n + 1 - i L H n + F H n + 1 L H n + i

= TLHnLHn+1 -|LHnFHn+1 +

| L^H
+ ^V(L?_
T
H n ^LHn ^
+i^TVV^Hn+1

- 4 ( - l ) H ^ ) / 5 V5(LL - 4 ( - l ) ^ )

Z JL
+ V ( Z * , - 4 ( - l ) H n + 1 ) ( Z nu - 4 ( - l ) H n )
n+l n
n+i
Now if H

= F , then Z = X and we get


n
n
n

s(R+l)
)(X2 - 4 ( - l ) s ( n ) )
n+2 ~ h Xn+l Xn + \f(X\ n+i - 4 { ~ l )
n

and if H = L , we have Z = W and


n
n
n
n
n+2

W
W + V & 2 T ^ 4 ( - l ) D^51+55^172
^ ' r x 0 ( W 2 - At
4 ( - li\sfaK
)OUi/)
n+l n
n+i
n

See p a g e 88 for R e f e r e n c e s .

(n > 0)
Using

SUMMATION OF S j | = 1 k Tk+r

FINITE DIFFERENCE

APPROACH

BROTHER ALFRED BROUSSEAU, St. Mary's College, California

Let it be proposed to discover an expression for the summation

! > %
k=i

or more generally

where m and r are positive integers. One possible approach is a modified


version of finite differences.

Given an expression f(n) where n is a positive

integer, the usual finite difference relation is


Af(n) = f(n 4- 1) - f(n)
The adapted finite difference pertains to a quantity of the form

f n

[ - V)]

where f is a function of n and Fibonacci numbers involving n in their subscripts. We shall define
Af

[n'V|=

f (n+1) F

' (n + l)]- f [ n ' F (n)]

For example,
A(n 2 F n ) = (n + l ) 2 F n + 1 - n 2 F n
= n 2 F n _i + (2n + l ) F n + 1
91

SUMMATION OF 2 ^ k m F k +

92

[Feb.

Likewise we define A~ to be the inverse of A so that


A_1 n2F

n-i

(2n + D F n + 1 ] = n 2 F n + C

where there is an arbitrary summation constant C which may involve Fibonacci


numbers but these as well as other constituent elements must be free of n.
For our purposes it turns out to be more convenient to seek the value of
n-i

k FT ,
k+r

E
k=i

Let this summation be denoted by $ [ n , F / n \ ] ,


n

Then

n-l

A4>[n,F (n) ] = X ) k H l F k + r " E ^ k + r


k=i

k=i

Thus
^n'

X/^k+r

^"'(^Fn+r)

k=i

We need then simply to evaluate this inverse finite difference in order to obtain
an expression for the summation.
We develop certain relations for this purpose.
(1)

(2)

A ( n F n + r + 1 ) = (n + l ) F n + r + 2 - n F n + r + 1 = n F n + r + F n + r + 2

A(n 2 F n + r + 1 ) = n 2 F n + r + (2n + l ) F n + r + 2 = n 2 F n + r + A(n 2 )F n + r + 2

and in general

(3)

A(n m F n +r+i) = n m F n + r + A(nHi)F n + r + 2

1967]

FINITE DIFFERENCE APPROACH

93

Using formula (1)


(4)

A ( n F n + r ) = n F n + r + 1 - A (F n + r + 2 ) = n F n + r + 1 - F n + r + 3 + C .

Then from this result and (2)


A ~ V F n + r ) = n 2 F n + r + 1 - (2n + l ) F n + r + 3 + 2 F n + r + 5 + C
= n 2 F n + r + 1 - A(n2)F n + r + 3 + A 2 (n2)F n + r + 5 + C
The general formula that suggests itself is
(6)

A _ 1 (nniF n + r ) = n m F n + r + 1 - A(nm)F n + r + 3 + A 2 (nm)F n + r + 5 + . . .


m

= J]' ( " 1 ) t A t ( n m ) F n + r + 2 t + 1 +
t=0
That this result is correct may be shown by calculating

AtA-Wn+r)]
from the summation in (6). The result is n m F n + r as can be readily seen from
the fact that apart from the first term in the expansion all succeeding terms
cancel in pairs.

The results for the first two terms will show the pattern,

A ( n m F n + r + 1 ) = n H i F n + r + A(nm)F n + r + 2

by (3)

AA(nm)F n + r + 3 ] = -A(n + l ) m F n + r + 4 + A ( n m ) F n + r + 3
= -A(nm)F n + r + 4 - A 2 ( n ^ ) F n + r + 4 + A(nm)F n + r + 3
= -A(nm)Fn+r+2 - A2(nm)Fn+r+4

Hence (6) provides the required formula apart from making explicit the coefficients in terms of n and calculating the undetermined constant. The former
are given subsequently in tables; the latter may be obtained as shown below for
the particular case in which m = 5.
We set n = 2 in (6) so that

94

kmFk+r

SUMMATION OF 2 ^

[Feb.

F r + 1 = 3 2 F r + 3 - 2 1 1 F r + 5 + 5 7 0 F r + 7 - 7 5 0 F r + 9 + 480 F r + 1 1 - 1 2 0 F r + 1 3 + C
Using the f o r m u l a s
Fn -

Fk+1Fn_k +

F ^ . ^

and
= (-D^FfeFn+fcH - Fk+1Fn+k)

C i s found to be 16679 F r + 9 + 10324 F r +

Table 1
COEFFICIENTS O F A ( n m )
m

n2

n4

n3

n5

n6

n8

n7

10

10

15

20

15

21

35

35

21

28

56

70

56

28

36

84

126

126

84

36

10

10

45

120

210

252

210

120

n5

n*

9
45

10

Table 2
COEFFICIENTS OF A 2 (nm )
m

n2

n4

n3

n7

14

24

30

70

60

20

62

180

210

120

30

126

434

630

490

210

42

254

1008

1736

1680

980

336

56

510

2286

4536

5208

3780

1764

504

72

5100

;11430

15120

13020

7560

2940

720

10

1022

12

90

19671

FINITE D I F F E R E N C E APPROACH

95

Table 3
COEFFICIENTS O F

n4

n3

A3(nm)

n5

n*

36

24

150

180

60

540

900

540

120

7
8

1806

3780

3150

1260

210

5796

14448

15120

8400

2520

336

18150

52164

65016

45360

18900

4536

504

10

55980

181500

260820

216720

113400

37800

7560

720

Table 4
4
COEFFICIENTS O F A (nm)

n2

24

240

120

1560

1440

360

8400

10920

5040

n5

n4

n3

840

40824

67200

43680

13440

1680

186480

367416

302400

131040

30240

3024

10

818520

1864800

1837080

1008000

327600

60480

5040

Table 5
COEFFICIENTS OF A 5 ( n m )

m
5

n2

n3

n4

120

1800

720

16800

12600

2520

126000

134400

50400

6720

834120

1134000

604800

151200

10

5103000

8341200

5670000

2016000

15120
378000

30240

SUMMATION O F S ^ = 1 k m F k + r

96

Table 6
COEFFICIENTS O F

[Feb.

A 6 (n m )

n2

n3

720

15120

5040

191520

120960

20160

1905120

1723680

544320

60480

10

16435440

19051200

8618400

1814400

Table 7
COEFFICIENTS OF
m

151200

A 7 (n m )
n2

n3

5040

141120

40320

2328480

1270080

181440

10

29635200

23284800

6350400

604800

Table 8
COEFFICIENTS OF A 8 ( n m )

m
8

n'

40320

1451520

362880

10

30240000

14515200

1814400

Table 9
COEFFICIENTS O F A 9 (n m )

n
362880

10

16329600

3628800

Table 10
COEFFICIENTS O F A 1 0 ( n m )
m
10

3628800

1967]

FINITE DIFFERENCE APPROACH

97

Table 11
SUMMATION CONSTANTS
m
1
2
3
4
5
6
7
8
9
10

Summation Constants
F

r+3
-F
r+6
7F , + 5 F ,
r+5
r+4
-37 F , - 24 F ,
r+6
r+5
242 F _ + 147 F ^
r+7
r+6
-1861 F ^ o - 1139 F L^
r+8
r+7
16679 F ^ + 1 0 3 2 4 F i o
r+9
r+8
- 1 7 1 3 6 2 F , - 106089 F ,
r+10
r+9
1981723 F _ + 1224729 F ^ 4 A
r+ii
r+io
-25453505 F ^ - 15726832 F ,
r+i2
r+11

To be able to w r i t e out a complete formula one u s e s formula (6) and the


various tables.

The c a s e m = 7 i s given below.

k7 FT ^ = n 7 F ^ _^ - v(7n6 + 21 n 5 + 35 n 4 + 35 n 3 + 21 n 2 + 7n + 1)F , ^
k+r
n+r+i
n+r+3
k=i

+ (v 4 2 n 5 + 2 1 0 n 4 + 4 9 0 n 3 + 630n 2 + 4 3 4 n + 126) F , ,
n+r+5
- ( 2 1 0 n 4 + 1260n 3 + 3150n 2 + 3 7 8 0 n + 1806) F , L
v
/ n+r+7
+ (840 n 3 + 5040 n2 + 10920 n + 8400)F ^ _,_
^
n+r+9
- (2520 n2 + 12600 n + 16800) F . , .. + (5040 n + 15120)F , ,. Q
n-i r+ 11
n+r+iu
- 5040 F ^ ^
n+r+15
CALCULATION BY FINITE D I F F E R E N C E S
Except for the s m a l l e r v a l u e s of m ,

the explicit f o r m u l a s given above

in t e r m s of n a r e apt to involve undue c a l c u l a t i o n s . T h e s e can be obviated by


going d i r e c t l y to finite differences and using f o r m u l a (6).
F o r e x a m p l e , to c a l c u l a t e

k5 F
*k+7

we would f i r s t w r i t e down the v a l u e s of k 5 for k = 50, 5 1 , e t c .

98

SUMMATION OF 2 **

k m F,

[Feb.

FINITE DIFFERENCE APPROACH

50

312500000

51

345025251

52

380204032

53

418195493

54

459165024

55

503284375

Then
A [ k 5 ] k = 5 0 = 34502521 - 312500000 = 32525251
A2 k5

[ ]k=50

A3

380204032

34502521 + 312500000 = 2653530

L k 5 ] k ^ 5 0 = 418195493 - 3 380204032 + 3 - 345025251 - 312500000


= 159150

A4 k

[ ]k=

459165024

"

' 418195493 + 6 380204032 - 4 345025251


+ 312500000 = 6240

A 5 [k 5 ]k=50

120

The value of the s u m m a t i o n i s :


312500000 F 5 8 - 32525251 F 6 0 + 2653530 F 6 2 - 159150 F 6 4 + 6240 F 6 6 - 120 F 6 8
+ 242 F 1 4 + 147F 1 3
which c a n e i t h e r be calculated d i r e c t l y o r s u m of the t e r m s c a n be unified and
the n u m b e r of multiplications of l a r g e n u m b e r s c a n be d e c r e a s e d .

* * *

ON RATIOS OF FIBONACCI AND LUCAS NUMBERS


G . F. Feeman, Williams C o l l e g e , W i l l i a m s t o w n , Massachusetts

Recently the author has conducted in-service training sessions in mathematics for the elementary school teachers of the Williams town, Massachusetts
public schools,, During a session on the lowest common multiple and greatest
common divisor of two positive integers, two teachers observed that if the two
numbers are in the ratio 2:3, then the sum of the numbers is equal to the difference between their lowest common multiple and their greatest common divisor.

It is shown in [2] that this is the only ratio for which this relation holds.
Of course, one gets similar relations for other ratios.

For example, if

the two numbers are in the ratio 3:5, then twice their sum is equal to the sum
of their lowest common multiple and their greatest common divisor.

Again it

is shown in [2] that this is the only ratio for which this relation holds. This is
not always the case since, for example, both ratios 5:7 and 4:11 yield the
result that three times the sum of the numbers is equal to the sum of their lowest common multiple and their greatest common divisor.
If one specializes to the Fibonacci and the Lucas sequences, one gets
theorems of the type given below, in which families of such relations are exhibited and formulas for finding all ratios satisfying these relations a r e obtained.
Let {F n } be the sequence of Fibonacci numbers, where F t = 1, F 2 = 1
and F n + 2 = F n + F n + 1 for n > 1.'
Let {LJQ} be the sequence of Lucas numbers, where
and 1^+2 = L n + Ln+i for n ^ 10
The following known results are assumed.

L 1 = 1, L2 = 3

(See [1] or [ 3 ] , J

(i)

Neighboring Fibonacci numbers are relatively prime.

(ii)

F n + 1 = FnFn+2 + (-l)n .

(iii)

F 2n _i = F n F n + 1 - F n _ t F n _ 2

(iv)

Neighboring Lucas numbers are relatively prime.

(v)

F2n = FnLn

(vi)

Ln+1 = F n + Fn+2 .
99

100

ON RATIOS OF FIBONACCI
For the remainder of th^ article, let

[Feb.

a and b be natural numbers.

Denote by [ a , b ] the lowest common multiple of a and b and by (a,b) the


greatest common divisor of a and b.
Theorem 1; (1) If a and b are in the ratio F n : F n + 1 , then
F n _ t (a + b) = [ a ? b ] + (-l) n (a,b)

for n >

2.

(2) Let c and d be relatively prime natural numbers with


b = (c/d)a.

If Fn_1(a + h) = [ a , b ] + (-l) n (a,b) for n > 3, then the number of

solutions for the ratio cid is one-half the number of divisors of F- n - 2 F n ,

and

among the solutions is the ratio F n : F n + 1 .


Proof: (1) Suppose b = ( F n / F n + 1 ) a .
and [ a , b ] = F n F n+1 k,

Then a = F n+1 k,

for k a natural number.

b = F n k, (a,b) = k

Then

F n + 1 (a + b) = F n ^(F n -f F n+1 )k = F n . 4 F n ^ 2 k = (F n + 1 - F n )F n + 2 k
= FttH(Fn+Fn+1)k - FnFn+2k
=

n F n+ik + (F2n+i - F n F n + 2 ) k

= [ a f b ] + (-l) n (a,b)
(2) If b = (c/d)a,

for n > 2 .

where c and d are relatively prime, then a = dk,

b = ck, (a,b) = k and [ a , b ] = cdk, for k a natural number.


F

a-i(

+ b

>

[ a > b ] + (-l) n (a,b)

Then

for n > 3

implies

F ^ f c + d)= cd + (-l) n

for which we wish to find all positive integral solutions. Solving for c, we get

I96t]

AND LUCAS NUMBERS


Fn_lCi - ( - l ) n
c

Fl_t
=

101

Fn_t

(-if

n-i

d -

Fn_{

s o that by (ii),

11-2-^11

c = Fn-i

We need T '

d -

"- <;,>i the c;ase d > ^

Fn^

v-i

, then c < 0.
n~i5

The totsj : iu . i b r

.- ' . a e ^ ; -i roUifew^ a ^

o and d i s given by the n u m b e r of

divisors of

,,

v ir /

that if c - A,

rf

8^> f.i^f
P ^

\ f

. for if 0 < d < F

J ap obvioi o symmetry in these solutions

so

i ' ^TLU V)i\ GO is c = B, d = A. Thus the n u m -

b e r of distinct nc<)vil'r V
'- * r?-i j ?-d is one-half the n u m b e r of d i v i s o r s of
^n-2^n*
Fiu?v?y. iC rl ~ <lH t
ii-3-

<1

n-2-^n
=
x

and the r a t i o

Fn:FT1+1

E x a m p l e : If n = 8,

n-i

Fn_2

i s among the solutions,

t h e n ' F n _ 2 = 8,

F n - 1 = 13,

(1) If a and b a r e in the ratio 21:34,

F n = 21, and F n + 1 = 3 4 .

then

13(a + b) = [ a , b ] + (a 5 b)
(2) If b = (c/d)a,

This c o m p l e t e s the proof,

then 13(a + b) = [ a , b ] + ( a , b )

implies that

168
c = 13 + d - 13

168 has 16 d i v i s o r s , so there are 8 distinct solutions.

They are: 14:181,

15:97, 16:69, 17:55, 19:41, 20:37, 21:34,

among which i s the

and

25:27,

Fibonacci pair 21:34. '


The following l e m m a i s needed for the proof of the second theorem.

102

ON RATIOS OF FIBONACCI

f Feb.

Lemma: F 2 n _ 1 = F j - ^ L ^ - L n L n + 1 for n ^ 2.
Proof: The proof is by induction.

The identity is easily verified for n = 2.

Assume it is true for n = k, so that


F

2k-i

k+i L k+2 "

k L k+l

Then
F

2k+i

2k

kLk

2k-i

( F k+2 -

kLk
F

k+i L k+2 -

k) L k+2 -

= F k L k + Fk+2(Lk+3 "

kLkH

kLk+i

k+i) "

kLk+2 "

kLk+i

k+2 L k+3 "

k+2 L k+i "

k L k + i - ( L k+2 -

k+2 L k+3 "

k+i L k+2

k + i ( L k + i ~ F k+2 "

k+i)Lk+i
F

k)

But
L

k+i - Fk+2 ~ F k

by (vi), so that
F

2k+i

k+2Lk+3 " L k+i L k+2

completing the induction step and the proof.


Theorem 2: (1) If a and b are in the ratio L n :L n + 1 ,
F n + 1 (a + b) = [ a , b ] + F 2 n ^(a,b)

then

for n > 2 .

(2) If a and b are in the ratio F n _ 2 : F n - 1 , then


F

n+i( a

+ b

>

[a>b]

2n~i(a>b) for n ^ 3 .

(3) Let c and d be relatively prime natural number s with

1967]

AND LUCAS NUMBERS

103

b = (c/d)a. If F n + 1 (a + b)= [a 9 b ]+ F 2n _ 1 (a,b) for n s 2, then the possible


ratios c:d are determined from the divisors of
these ratios is Lj^Lft-fj*

For n 3,

(F^+j - F2n-.1)

Among

n-2* n--i ^ also a solution.

Proof: (1) Suppose


b = zr a
MB+i

Then
a = L n+1 k,

b = L n k,

for k a natural number.

(a,b) = k

and

[a,b] = L n L n + 1 k

Then

F n + i (a,b) = F n + 1 (L n + L n + 1 )k = F n + 1 L n + 2 k

Using the lemma, we get


F n + 1 (a + b) = (F2n-i + L n L n + 1 )k

so that

F n + 1 (a + b)= [a,b]'+ F 2n 1 (a,b)

as required.
(2) If

n-2

n-i

b =

then
a

n-ik >

for k a natural number.

n-2k >

(a>)

Then, using (iii),

and

[a>b]

"

n-iFn-2k

104

ON RATIOS OF FIBONACCI
F n + 1 (a + b) = F ^ F ^
=

( F 2n-i

[Feb.

+ F n _ 2 )k F

Fn+1Fnk

n-iFn-2)k

= [ a , b ] +. F ^ a . b )

as required.
(3) If b = (c/d)a, where c and d are relatively prime, then, once
again
a = dk,

b = ck,

for k a natural number,

(a,b) = k

and

[ a , b ] = cdk

The relation

F n + 1 (a + b) = [ a , b ] + F 2n i(a 9 b)
implies
F n+1 (c + d) cd + F 2 I W
Solving this equation for c, we get

F n + 1 d - F 2 n -i
_ _ _ _ _

^
=

Fn+l

F n + 1 - F2n_1
^ - ^ r ^

We seek positive integral solutions for c and d.

The possible ratios c:d

are determined from the divisors of (Fn+t - F 2n _ 1 ) .


Using the lemma, we show that c = L n , d = L n + 1 is a solution.
symmetry, c = L n + j ,

d = L n is also a solution.

n+iLn+i ~ F 2n-i _

n+i L n+i ~ F n+i L n+2

Ln+i "" F n+i


F

So let d = L n + 1 .

n+i(Ln+i - Ln+2) + L n L n+i

Ln+i ~ F n+i

Then

n L n+l

n+i ~ F n+i
"LnFn+i

By

n L n+i

n+i ~ F n+i

= Ln

1967 ]

AND LUCAS NUMBERS

105

The situation here differs from that in the second part of Theorem 1,
for not all solutions are obtained by considering the case d >F n + lo
example, let d = Fn_lffl
F
Q

Then s using (iii),

n+iFn-i -

- F n+i F n-2

Thus the ratio


theore m

n-2

:F

n-i -

For

2 n-i
:

n*l

=T

n-i - FnFn+1 + Fn-1Fn2

, .

n+i

n-i

n-iFn-2
" ^
"

n-i

*s

n-i -

n+i

rn

solution,,

~2
This completes the proof of the

Example: If n = 7, then L n = 29, L n + 1 = 47, F n _ 2 = 5, Fnt

= 8, F n + 1

= 21 and F 2 n -i = 233.
(1) and (2): If a and b are in the ratio 29:47 or 5:8, then
21 (a + b) = [a s b] + 233(a,b)
(3): If b = (c/d)a,

then

21(a + b ) ' = [a,b] + 233(a,b)


implies that
c

21 +

M1^|33
d - 21

2 1 +

"

208
d - 21

The divisors of 208 are 1, 2, 4, 8, 13, 16, 26, 52, 104 and 208.

The solu-

tions are 22:229, 23:125, 25:73, 29:47, 34:37 and 5:8. Among these ratios
are 29:47 = L ^ L ^ i ' and 5:8 = F ^ i F ^ .

ACKNOWLEDGEMENTS
The final version of this article was written while the author held a National
Science Foundation Science Faculty Fellowship. The author wishes to thank the
reviewer and Professor V. E. Hoggatt, J r . , for their helpful suggestions and
comments.

106

ON RATIOS OF FIBONACCI AND LUCAS NUMBERS

Feb. 1967

1. S. L. Basin and V. E. Hoggatt, J r . , "A Primer on the Fibonacci Sequence,


Part I I , " Fibonacci Quarterly, Vol. 1, No. 2 (1-963), pp 61-68.
2. G. Cross and H. Renzi, "Teachers Discover New Math T h e o r e m / ' The
Arithmetic Teacher, Vol. 12 No. 8 (1965), pp 625-626.
3. G. H. Hardy and E. M. Wright, An Introduction to the Theory of Numbers,
Oxford, London, 1954, pp 148-150.
*

All subscription correspondence should be addressed to Brother U8 Alfred,


St. Mary f s College, Calif. All checks ($4.00 per year) should be made out to
the Fibonacci Association or the Fibonacci Quarterly.

Manuscripts intended

for publication in the Quarterly should be sent to Verner E. Hoggatt, J r . ,


Mathematics Department, San Jose State College, San Jose, Calif. All manuscripts should be typed, double-spaced. Drawings should be made the same size
as they will appear in the Quarterly, and should be done in India ink on either
vellum or bond paper. Authors should keep a copy of the manuscripts sent to
the editors.
*

NOTICE TO ALL SUBSCRIBERS!!!


Please notify the Managing Editor AT ONCE of any address change. The Post
Office Department, rather than forwarding magazines mailed third class, sends
them directly to the dead-letter office.

Unless the addressee specifically r e -

quests the Fibonacci Quarterly to be forwarded at first class rates to the new
address, he will not receive it. (This will usually cost about 30 cents for firstclass postage.) If possible, please notify us AT LEAST THREE WEEKS PRIOR
to publication dates: February 15, April 15, October 15, and December 15.
*

ELEMENTARY PROBLEMS AND SOLUTIONS


Edited by A . P H I L L M A N , University of New M e x i c o , Albuquerque, New M e x .

Send all communications r e g a r d i n g E l e m e n t a r y P r o b l e m s and Solutions


to P r o f e s s o r A. P . Hillman, D e p a r t m e n t of M a t h e m a t i c s and S t a t i s t i c s ,
v e r s i t y of New Mexico, Albuquerque, New Mexico 87106.

Uni-

Each p r o b l e m o r

solution should be submitted in legible form, p r e f e r a b l y typed in double spacing,


on a s e p a r a t e sheet o r s h e e t s in the format used below.

Solutions should be

r e c e i v e d within t h r e e months of the publication d a t e .


B-106 Proposed by H. H. Ferns, Victoria, B . C . , Canada.
P r o v e t h e following identities*
2 F . . . = F.L. + F.L. ,
i+3
13
3 1
2 K . = L.L. + 5F.F. .
1+3

1 3

1 3

B-107 Proposed by Robert S. Seamons, Yakima Valley College, Yakima, Wash.


Let M

and G

b e r e s p e c t i v e l y the n

t e r m s of the s e q u e n c e s

L u c a s and Fibonacci) for which M n = M^_t - 2,


G o55
n-2

Gil = 1,

Go
L = 2e

Mi = 3,

(of

and G n = G ^ +

P r o v e that

Mn = 1 + K
L V/ 5 G
m J1 ,
w h e r e m = 2 n - 1 and [ x ] i s t h e g r e a t e s t integer function,,
B-108 Proposed by V . E. Hoggatt, J r . , San Jose State College, San Jose, Calif.
Let Ui = p ,
+u .

y 2 = q,

and u n + 2 = u n + 1 + u n o Also l e t S n = Ui + u 2 +

It i s t r u e that S6 = 4u 4 and S10 = l l u 7 .

Generalize these formulas.

B-109 Proposed by V . E. Hoggatt, J r . , San Jose State College, San Jose, Calif.
Let r and s b e t h e r o o t s of the q u a d r a t i c equation
(r f s ) .

Let U

= (r11 - s n ) / ( r - s) and V

= rn + sn.

V = U _, + qU ,
n
n+i
^ n-l
107

x 2 - px - q = 05

Show that

108

ELEMENTARY PROBLEMS AND SOLUTIONS

[Feb.

B-110 Proposed by L. Carlitz, Duke University, Durham, N . Carolina.


Show that

oo

oo

=V5V<^

X4 r 2 n + i

n=o

-*> L 2 n + i

n=o

B-111 Proposed by L. Carlitz, Duke University, Durham, N . Carolina.


Show that

J F 4 n + 2

oo

~* L 4n+2

SOLUTIONS
LUCAS NUMBERS MODULO 5
B-88 Proposed by John Wessner, Melbourne, Florida.

Let L0, L2, L4, Lg, be the Lucas numbers 2, 3, 7, 18, .

Show

that
L2k = 2 ( - l ) k

(mod 5)

Solution by J . A . H. Hunter, Toronto, Canada

All (mod 5) we have: LA = 1, L2 = - 2 , L3 = - 1 , L4 = 2, L 5 = 1, L6 = - 2 , .


etc., so it follows that L4t+2 = -2 and L ^ = +2. Hence L2k = 2(-l) (mod 5).
Also solved by James E. Desmond, H. H. Ferns, Joseph D. E. Konhauser,
Douglas Lind, F. D. Parker, C . B . A . Peck, Jeremy C. Pond, D a v i d Z e i t l i n ,
and the proposer.

A CLOSE APPROXIMATION
B-89 Proposed by Robert S. Seamons, Yakima Valley College, Yakima, Wash.

Let Fn and Ln be the n Fibonacci and n Lucas numbers,>respectr


ively. Let [x] be the greatest integer function. Show that L 2 m = 1 +
[\/5F 2m ] for all positive integers m.
Solution by Douglas Lind, University of Virginia, Charlottesville, V a .

0,2m +

From the Binet forms for F and L , the statement is equivalent to


n
n
2m
2m
2 m = [1 + a
- ] , where a = (1 + V5)/2, j8 = (1 --V5)/2. But

1/2 > / 3 2 m > 0 for m > 0, so we have

1967]

ELEMENTARY PROBLEMS AND SOLUTIONS


^ m + 2m < ^2m ^2111

<

a2m

which i m p l i e s a2111 + / 3 2 m = [ l + # 2 m - / 3 2 m ] ,

^m

109

as desired.

Also solved by James E. Desmond, H. K-. Ferns, C . B . A . Peck, Jeremy C Pond,


David Z e i t l i n , and the proposer*

B-90 Proposed by Phil Mana, University of New Mexico, Albuquerque, N . Mex,


Let b j , b 2 , b e the sequence 3, 7, 47, with r e c u r r e n c e r e l a t i o n
b , A = b 2 - 2.
n+i
n

Show that the r o o t s of


x 2 - 2b x + 4 = 0
n

a r e e x p r e s s i b l e in the form c + dV5, w h e r e c and d a r e i n t e g e r s .


Solution by David Z e i t l i n , Minneapolis, Minnesota.
The r o o t s a r e b \ / b 2 - 4 = b x/b
,, - 2. The r e c u r s i o n r e l a t i o n
n
n
n v n+i
may b e w r i t t e n a s U n + 1 = (b n + 2)U n , w h e r e U n = b n - 2, U 4 = 1. T h u s ,
U

ni

XL,

_*i

k=i

n
=

k=i

(b, + 2) = 5 n (bk + 2)
k

k=2

n-i
n-i
= 5 n ( b . + 1 + 2) = 5 n b 2
j=i

or

j=i

n-i
b

- 2 = 5 FI b 2
j=1 3

Thus,
c = b ,

and

n-i
d = II b . ,

n = 2,3,

Also solved by James E. Desmond, H H Ferns, Douglas Lind, O B . A * Peck,


Jeremy C, Pond, John Wessner, and the proposer*

110

ELEMENTARY PROBLEMS AND SOLUTIONS

[Feb.

CONVERGENCE OF SERIES
B-91 Proposed by Douglas Lind, University of Virginia, Charlottesville, V a ,

If F

is the n

Fibonacci number, show that

I>/ F j>
i=i

converges while

Z(1/lnFj)
j=3

diverges.
Solution by Jeremy C* Pond, Sussex, England*
F
n+l
=
n

^ 1 +\/5
v
> 1 as

and so

converges by d'Alembert's test.

Also,

(l/lnFn)/(l/n)-^l/ln(^-i^j
and so 1/ln F. and 1/n diverge together.
Also solved by C* B. A, Peck and the proposer*

> 0

n (

1967]

ELEMENTARY PROBLEMS AND SOLUTIONS

111

GREATEST COMMON DIVISOR


B-92 Proposed by J * L. Brawn, Jr*, The Pennsylvania State University,
L e t (x,y) denote the g c d of positive i n t e g e r s x and y.
(F
v

Show that

,F ) = (F 5, F , ) = (F , F , ) for all positive i n t e g e r s m and n


m5 n
m m+n
n m+n
.

I, Solution by Joseph D. E. Konhauser# Univ. of Minnesota, Minneapolis, Minn,


We u s e the well-known identity
F

= F
m+n

F
+ F F
n-l m
n m+i

and the fact that two consecutive Fibonacci n u m b e r s a r e r e l a t i v e l y p r i m e .


Let d = (F

, F ) then, from (1) d | F


. Let e = (F , , F ) then,
m n
m+n
m+n
m
from (1), e | F , s i n c e (F , F , J = .1. On the one hand., eld (since e | F
v /?
x
s v
| n?
m9 m+r
i m
and e |IFn 7). On the other hand,> die
d ||F m and d || F m + ,n ' ). T h e r e f o r e,
i (since
\
d = e; that I s , (F , F ) = (F , F , ). In like m a n n e r . It follows that
' v m' n
m ? m+n
'
(F
x
?, F 7) = (F J, F
7k
m n
n m+n
II.

Solution by Douglas Lind, University of Virginia, Charlottesville, Virginia.


It i s well known [N. N8 Vorobyov, The Fibonacci N u m b e r s ,

page 2 3 ,

T h e o r e m 41J that v(F

, F ) = F,
* The d e s i r e d r e s u l t then follows I m m e d i m n
(m,n)x
ately from the easily e s t a b l i s h e d fact that (m,n) = (m,m + n) = (n,m + n).

Also solved by Thomos P. Dence, James E. Desmond, John E. Homer, J r . ,


C.B.A. Peck, Jeremy C. Pond, David Zeitlin, and the proposer.
L

MODULO n
n

B-93 Proposed by Martin Pettet, Toronto, Ontario, Canada


Show that If n i s a positive p r i m e , L

= 1 (mod n). Is the c o n v e r s e t r u e ?

Solution by Douglas Lind, University of Virginia, Charlottesville, Virginia*


F r o m the Binet f o r m we have

112

ELEMENTARY PROBLEMS AND SOLUTIONS

Feb. 1967

where [x] denotes the greatest integer contained in x. Now if n is prime,

(2j)=

(mod

> 0

l>2>'->

n / 2 ),

so that

\ 1 (S)/2""' = ^ '

>

By Fermat T s Lesser Theorem, 2 ~ = 1 (modn), so that L

= 1 (mod n) if

n is prime.
I have not been able to prove or disprove the converse of this statement.
A calculation by computer indicates that the converse is true for n < 700.
Also solved by the proposer who stated that the converse is false and gave 705,
2465, and 2737 as the first few composite values of n.

*****
NOTICE
George Ledin, J r . has been appointed by The Fibonacci Association to collect
and classify all existing Fibonacci Identities, Lucas Identities, and Hybrid
Identities. We request that readers send copies of their private lists (with
possible reference sources) to
George Ledin, J r .
445 Monticello
San Francisco, Calif. 94127
for inclusion in the planned booklet.
Verner E. Hoggatt, J r . ,
Director
*

You might also like